Executive Assitant : Date - 21/11/2022 Shift - 1 Previous Year Paper Full Length PDF


Question 1


Which of the following options is used to resize rows in MS-Excel 365?

MS-Excel 365 में पंक्तियों (rows) का आकार बदलने के लिए निम्न में से किस विकल्प का उपयोग किया जाता है?

Options

A

Format as Table -> Format -> Row Width or Row Height

Format as Table -> Format -> Row Width or Row Height

B

Home -> Cell Styles -> Row Width or Row Height

Home -> Cell Styles -> Row Width or Row Height

C

Home -> Format -> Row Width or Row Height

Home -> Format -> Row Width or Row Height

D

Home -> Insert -> Row Width or Row Height

Home -> Insert -> Row Width or Row Height


Solution:

Correct Answer:

C

Home -> Format -> Row Width or Row Height

होम -> फॉर्मेट -> रो विड्थ या रो हाइट
(Home -> Format -> Row Width or Row Height)


Question 2


Which of the following shortcut keys is used change the font size for selected text in MS-PowerPoint 365?

एमएस-पॉवरपॉइंट 365 में सेलेक्टेड टेक्स्ट के फॉन्ट साइज बदलने के लिए निम्नलिखित में से किस शॉर्टकट कुंजियों (keys) का उपयोग किया जाता है?

Options

A

Alt+N, S, H

Alt+N, S, H

B

Alt+W, Q

Alt+W, Q

C

Alt+H, F, S

Alt+H, F, S

D

Alt+G, H

Alt+G, H


Solution:

Correct Answer:

C

Alt+H, F, S

Alt+H, F, S


Question 3


MS-Word 365 के निम्नलिखित में से किस विकल्प का उपयोग सेलेक्टेड टेक्स्ट (selected text), पैराग्राफ (paragraph) या टेबल सेल (table cell) के पीछे का रंग बदलने के लिए किया जाता है?

MS-Word 365 के निम्नलिखित में से किस विकल्प का उपयोग सेलेक्टेड टेक्स्ट (selected text), पैराग्राफ (paragraph) या टेबल सेल (table cell) के पीछे का रंग बदलने के लिए किया जाता है?

Options

A

शेडिंग (Shading)

शेडिंग (Shading)

B

टेक्स्ट हाईलाइट कलर (Text Highlight Color)

टेक्स्ट हाईलाइट कलर (Text Highlight Color)

C

टेक्स्ट इफेक्ट्स एंड टाइपोग्राफी (Text Effects and Typography)

टेक्स्ट इफेक्ट्स एंड टाइपोग्राफी (Text Effects and Typography)

D

फॉन्ट कलर (Font Color)

फॉन्ट कलर (Font Color)


Solution:

Correct Answer:

A

शेडिंग (Shading)

शेडिंग (Shading)


Question 4


Which of the following is FALSE with respect to the control unit of a computer?

कंप्यूटर की नियंत्रण इकाई (control unit) के संबंध में निम्नलिखित में से कौन-सा तथ्य असत्‍य है?

Options

A

It is responsible for controlling the transfer of data and instructions among other units of the computer.

It is responsible for controlling the transfer of data and instructions among other units of the computer.

B

It obtains instructions from the memory, interprets them, and directs the operation of the computer.

It obtains instructions from the memory, interprets them, and directs the operation of the computer.

C

It processes and stores data.

It processes and stores data.

D

It manages and coordinates all the units of the computer.

It manages and coordinates all the units of the computer.


Solution:

Correct Answer:

C

It processes and stores data.

यह डेटा को प्रोसेस और स्टोर करती है।


Question 5


Which of the following is the core component of Linux that interacts with hardware?

निम्नलिखित में से कौन-सा, लिनक्स का वह मुख्य घटक है जो हार्डवेयर के साथ इंटरैक्ट (interact) करता है?

Options

A

Shell

Shell

B

Editor

Editor

C

GUI

GUI

D

Kernel

Kernel


Solution:

Correct Answer:

D

Kernel

कर्नेल (Kernel)


Question 6


The main memory of a computer is made using which of the following technologies?

कंप्यूटर की मुख्य मेमोरी निम्नलिखित में से किस तकनीक का उपयोग करके बनाई जाती है?

Options

A

Vacuum tube

Vacuum tube

B

Magnetic

Magnetic

C

Semi-conductor

Semi-conductor

D

Optical

Optical


Solution:

Correct Answer:

C

Semi-conductor

सेमी-कंडक्टर (Semi-conductor)


Question 7


Which of the following statements is/are true with reference to GUI based Operating Systems?

(i) Both DOS and Linux are GUI based.
(ii) Both DOS and Unix are GUI based.

GUI आधारित ऑपरेटिंग सिस्टम (Operating Systems) के संदर्भ में निम्नलिखित में से कौन-सा/से कथन सत्य है/हैं?

(i) डॉस (DOS) और लिनक्स (Linux) दोनों GUI आधारित होते हैं।
(ii) डॉस (DOS) और यूनिक्स (Unix) दोनों GUI आधारित होते हैं।

Options

A

Only (i)

Only (i)

B

Both (i) and (ii)

Both (i) and (ii)

C

Neither (i) nor (ii)

Neither (i) nor (ii)

D

Only (ii)

Only (ii)


Solution:

Correct Answer:

C

Neither (i) nor (ii)

न तो (i) न ही (ii)


Question 8


Linux is a portable operating system. Which of the following best describes the portability feature of Linux?

लिनक्स (Linux) एक पोर्टेबल ऑपरेटिंग सिस्टम (portable operating system) होता है। निम्नलिखित में से कौन-सा विकल्‍प लिनक्स के पोर्टेबिलिटी फीचर (portability feature) का सबसे अच्छा वर्णन करता है?

Options

A

Linux can support GUI.

Linux can support GUI.

B

Linux can work on different types of hardware.

Linux can work on different types of hardware.

C

Linux can support multiple users at the same time.

Linux can support multiple users at the same time.

D

Linux can support multiple programs at the same time.

Linux can support multiple programs at the same time.


Solution:

Correct Answer:

B

Linux can work on different types of hardware.

लिनक्स (Linux) विभिन्न प्रकार के हार्डवेयर पर कार्य कर सकता है।


Question 9


Which of the following options is NOT available when we click the ‘Delete’ option on the Mini toolbar that is displayed by right-clicking in a table cell, row, or column you want to delete in MS-Word 365?

MS-Word 365 में जब हम मिनी टूलबार (Mini toolbar), जो उन टेबल सेल, रो या कॉलम में राइट-क्लिक करने से प्रदर्शित होता है जिन्हें आप डिलीट करना चाहते हों, में 'डिलीट' (Delete) विकल्प पर क्लिक करते हैं, तो निम्नलिखित में से कौन-सा विकल्प उपलब्ध नहीं होता है?

Options

A

Delete Rows

Delete Rows

B

Delete Columns

Delete Columns

C

Delete Cells

Delete Cells

D

Delete Pages

Delete Pages


Solution:

Correct Answer:

D

Delete Pages

डिलीट पेजेज़ (Delete Pages)


Question 10


Which of the following is correct in terms of increasing order of access speed?

एक्सेस स्पीड (access speed) के बढ़ते क्रम के संदर्भ में निम्नलिखित में से कौन-सा विकल्प सही है?

Options

A

B

C

D


Solution:

Correct Answer:

A


Question 11


TCP is a/an ______ protocol, whereas UDP is a/an ______ protocol.

TCP एक ______ प्रोटोकॉल है, जबकि UDP एक ______ प्रोटोकॉल है।

Options

A

application-layer; connection-oriented

application-layer; connection-oriented

B

connection-oriented; connection-less

connection-oriented; connection-less

C

connection-less, connection-oriented

connection-less, connection-oriented

D

connection-less; application-layer

connection-less; application-layer


Solution:

Correct Answer:

B

connection-oriented; connection-less

कनेक्शन-ओरिएंटेड (connection-oriented); कनेक्शन-लेस (connection-less)


Question 12


Which of the following options can be used to quickly create a list of dates, in sequential order in MS-Excel 365?

MS-Excel 365 में अनुक्रमिक क्रम में दिनांकों की सूची को त्वरित रूप से बनाने के लिए निम्नलिखित में से किस विकल्प का उपयोग किया जा सकता है?

Options

A

Auto Run

Auto Run

B

Fill Handle

Fill Handle

C

Auto hide

Auto hide

D

Auto Complete

Auto Complete


Solution:

Correct Answer:

B

Fill Handle

फिल हैंडल (Fill Handle)


Question 13


Which of the following function keys is used to open the Help Viewer window anytime in MS-Word 365?

MS-Word 365 में किसी भी समय हेल्प व्यूअर विंडो (Help Viewer window) खोलने के लिए निम्नलिखित में से किन फंक्शन-कुंजियों (function keys) का उपयोग किया जाता है?

Options

A

F4

F4

B

F1

F1

C

F10

F10

D

F5

F5


Solution:

Correct Answer:

B

F1

F1


Question 14


Which of the following is the correct option to save a file in MS-PowerPoint 365?

एमएस-पॉवरपॉइंट 365 में किसी फाइल को सेव करने का सही विकल्प निम्नलिखित में से कौन-सा है?

Options

A

Insert -> Save

Insert -> Save

B

Draw -> Save

Draw -> Save

C

Design -> Save

Design -> Save

D

File -> Save

File -> Save


Solution:

Correct Answer:

D

File -> Save

फाइल (File) -> सेव (Save)


Question 15


Which of the following is used to refer to all cells in row 5 in MS-Excel 365 formula?

MS-Excel 365 फॉर्मूला में पंक्ति (row) 5 में सभी सेलों को संदर्भित करने के लिए निम्नलिखित में से किसका उपयोग किया जाता है?

Options

A

5

5

B

$5$5

$5$5

C

5$5$

5$5$

D

5:5

5:5


Solution:

Correct Answer:

D

5:5

5:5


Question 16


Which of the following internet services describes how to make and receive phone calls over the internet?

निम्नलिखित में से कौन-सी इंटरनेट सेवा, इंटरनेट पर फोन कॉल करने और प्राप्त करने का तरीका बताती है?

Options

A

VoIP

VoIP

B

Internet Relay Chat

Internet Relay Chat

C

USENET

USENET

D

e-mail

e-mail


Solution:

Correct Answer:

A

VoIP

वीओआईपी (VoIP)


Question 17


Which of the following can be used to change the Desktop background in a Windows 10 operating system?

किसी विंडोज़ 10 ऑपरेटिंग सिस्टम (Windows 10 operating system) में डेस्कटॉप बैकग्राउंड (Desktop background) को बदलने के लिए निम्नलिखित में से किसका उपयोग किया जा सकता है?

Options

A

Double click anywhere on the Desktop

Double click anywhere on the Desktop

B

Right click on Desktop and select the ‘Personalize’ option

Right click on Desktop and select the ‘Personalize’ option

C

Right click on Desktop and select the ‘New’ option

Right click on Desktop and select the ‘New’ option

D

Triple click anywhere on the Desktop

Triple click anywhere on the Desktop


Solution:

Correct Answer:

B

Right click on Desktop and select the ‘Personalize’ option

डेस्कटॉप पर राइट क्लिक करें और 'पर्सनलाइज़'(Personalize) ऑप्शन को सेलेक्ट करें


Question 18


Which of the following options best describes the truthfulness of the given statements?

(i) Type "WinWord.exe" without quotation marks in the Run dialogue box and click ‘OK’
(ii) Type "Word.exe" without quotation marks in the Run dialogue box and click ‘OK’

निम्नलिखित में से कौन-सा विकल्प, दिए गए कथनों की सत्यता का सर्वोत्तम वर्णन करता है?

(i) रन डायलॉग बॉक्स (Run dialogue box) में कोटेशन मार्क के बिना "WinWord.exe" को टाइप करें और ‘OK’ पर क्लिक करें
(ii) रन डायलॉग बॉक्स (Run dialogue box) में कोटेशन मार्क के बिना "Word.exe" को टाइप करें और ‘OK’ पर क्लिक करें

Options

A

(i)-True (ii)-False

(i)-True (ii)-False

B

(i)-False (ii)True

(i)-False (ii)True

C

(i)-True (ii)-True

(i)-True (ii)-True

D

(i)-False (ii)-False

(i)-False (ii)-False


Solution:

Correct Answer:

A

(i)-True (ii)-False

(i)-सत्‍य (ii)-असत्‍य


Question 19


Which of the following is true with reference to a Local Area Network?

लोकल एरिया नेटवर्क (Local Area Network) के संदर्भ में निम्नलिखित में से कौन-सा तथ्य सत्य है?

Options

A

It connects devices that are in a single, limited area.

It connects devices that are in a single, limited area.

B

It is a collection of devices connected in different countries.

It is a collection of devices connected in different countries.

C

WAN cannot connect many LANs together.

WAN cannot connect many LANs together.

D

LANs can be operated only on Client/Server technology.

LANs can be operated only on Client/Server technology.


Solution:

Correct Answer:

A

It connects devices that are in a single, limited area.

यह उन डिवाइसों को जोड़ता है जो किसी एकल, सीमित क्षेत्र में हों।


Question 20


Which of the following is NOT true to start working in Edit mode in MS-Excel 365?

MS-Excel 365 में एडिट मोड (Edit mode) में कार्य प्रारंभ करने के लिए, निम्नलिखित में से कौन-सा तथ्य सही नहीं है?

Options

A

Click the cell that contains the data that you want to edit, and then click anywhere in the formula bar

Click the cell that contains the data that you want to edit, and then click anywhere in the formula bar

B

Click the cell that contains the data that you want to edit, and then press F2

Click the cell that contains the data that you want to edit, and then press F2

C

Double-click the cell that contains the data that you want to edit

Double-click the cell that contains the data that you want to edit

D

Click the cell that contains the data that you want to edit, and then press F5

Click the cell that contains the data that you want to edit, and then press F5


Solution:

Correct Answer:

D

Click the cell that contains the data that you want to edit, and then press F5

उस सेल पर क्लिक करना जिसमें वह डेटा है जिसे आप संपादित करना चाहते हैं, और फिर F5 को दबाना


Question 21


Which of the following options of the Slide Show menu shows the current slide, the next slide, and speaker notes, to help us focus while presenting and to connect with the audience in MS-PowerPoint 365?

स्लाइड शो (Slide Show) मेनू का निम्नलिखित में से कौन-सा विकल्प ऐसे करेंट स्लाइड, नेक्स्ट स्लाइड और स्पीकर नोट्स को दिखाता है, जिससे हमें एमएस-पॉवरपॉइंट 365 में प्रेजेंटेशन के समय फोकस करने और दर्शकों के साथ जुड़ने में सहायता मिलती है?

Options

A

Show Media Controls

Show Media Controls

B

Use Timings

Use Timings

C

Use Presenter View

Use Presenter View

D

Play Narrations

Play Narrations


Solution:

Correct Answer:

C

Use Presenter View

यूज प्रेजेंटर व्यू (Use Presenter View)


Question 22


Which of the following options in MS-Word 365 is used to set up layout and print options of a page?

MS-Word 365 में निम्नलिखित में से कौन-सा विकल्प, किसी पेज के लेआउट और प्रिंट विकल्पों को सेट करने में उपयोग किया जाता है?

Options

A

Page Setup

Page Setup

B

Review

Review

C

Design

Design

D

Draw

Draw


Solution:

Correct Answer:

A

Page Setup

पेज सेटअप (Page Setup)


Question 23


Which of the following is NOT a characteristic of a computer?

निम्नलिखित में से कौन-सी, कंप्यूटर की एक विशेषता नहीं है?

Options

A

Accuracy

Accuracy

B

Speed

Speed

C

Versatility

Versatility

D

Intelligent quotient

Intelligent quotient


Solution:

Correct Answer:

D

Intelligent quotient

बुद्धि लब्धि (Intelligent quotient)


Question 24


Which of the following file formats is used to save a workbook as a comma-delimited text file for use on another Windows operating system in MS-Excel 365?

MS-Excel 365 में किसी अन्य विंडोज़ ऑपरेटिंग सिस्टम पर उपयोग किए जाने के लिए, किसी वर्कबुक को कॉमा-डिलिमिटेड (comma-delimited) टेक्स्ट फाइल के रूप में सेव (save) करने में निम्नलिखित में से किस फाइल फॉर्मेट का उपयोग किया जाता है?

Options

A

Formatted Text

Formatted Text

B

CSV (comma delimited)

CSV (comma delimited)

C

Text (MS-DOS)

Text (MS-DOS)

D

Unicode Text

Unicode Text


Solution:

Correct Answer:

B

CSV (comma delimited)

सीएसवी (कॉमा डिलिमिटेड) CSV (comma delimited)


Question 25


Which of the following is the correct way to create a presentation using Templates in MS-PowerPoint 365?

एमएस-पॉवरपॉइंट 365 में टेम्प्लेट (Template) का उपयोग करके प्रेजेंटेशन बनाने का सही तरीका निम्नलिखित में से कौन-सा है?

Options

A

File -> Open -> Search for online templates and themes

File -> Open -> Search for online templates and themes

B

File -> Recent -> Search for online templates and themes

File -> Recent -> Search for online templates and themes

C

File -> Home -> Search for online templates and themes

File -> Home -> Search for online templates and themes

D

File -> New -> Search for online templates and themes

File -> New -> Search for online templates and themes


Solution:

Correct Answer:

D

File -> New -> Search for online templates and themes

फाइल (File) -> न्यू (New) -> सर्च फॉर ऑनलाइन टेंपलेट्स एंड थीम्स (Search for online templates and themes)


Question 26


Which of the following is NOT a browser software?

निम्नलिखित में से कौन-सा, एक ब्राउज़र सॉफ्टवेयर नहीं है?

Options

A

DuckDuckGo

DuckDuckGo

B

Microsoft Edge

Microsoft Edge

C

Brave

Brave

D

Chromium

Chromium


Solution:

Correct Answer:

A

DuckDuckGo

डकडकगो (DuckDuckGo)


Question 27


Which of the following is true with reference to Print Preview in MS-Word 365?

MS-Word 365 में प्रिंट प्रीव्यू (Print Preview) के संदर्भ में निम्नलिखित में से कौन-सा कथन सत्य है?

Options

A

Click File > Print, On the right, you’ll see the Print button and the configurable Settings.

Click File > Print, On the right, you’ll see the Print button and the configurable Settings.

B

Click File > Print, On the left, you’ll see a preview of your presentation.

Click File > Print, On the left, you’ll see a preview of your presentation.

C

In the paragraph group of the Home tab, we can choose options such as printing only certain pages, changing the page orientation etc.

In the paragraph group of the Home tab, we can choose options such as printing only certain pages, changing the page orientation etc.

D

Word combines Print and Preview in the same window.

Word combines Print and Preview in the same window.


Solution:

Correct Answer:

D

Word combines Print and Preview in the same window.

वर्ड (Word) एक ही विंडो में प्रिंट (Print) और प्रीव्यू (Preview) को जोड़ता है।


Question 28


Which of the following symbols is used to insert a smiley emoji in e-mail in Outlook 2016?

आउटलुक 2016 में ई-मेल (e-mail) में स्माइली इमोजी (smiley emoji) डालने के लिए निम्नलिखित में से किस प्रतीक का उपयोग किया जाता है?

Options

A

;-)

;-)

B

:-)

:-)

C

:-O

:-O

D

:-(

:-(


Solution:

Correct Answer:

B

:-)

:-)


Question 29


Which of the following option is used to set the transition speed of a slide in MS-PowerPoint 365?

एमएस-पॉवरपॉइंट 365 में स्लाइड की ट्रांजिशन स्पीड (transition speed) को सेट करने के लिए निम्नलिखित में से किस विकल्प का उपयोग किया जाता है?

Options

A

Animations -> Duration

Animations -> Duration

B

Transition -> Duration

Transition -> Duration

C

Slide Show -> Duration

Slide Show -> Duration

D

Design -> Duration

Design -> Duration


Solution:

Correct Answer:

B

Transition -> Duration

ट्रांजिशन (Transition) ->ड्यूरेशन (Duration)


Question 30


Which of the given options best describes the truthfulness of the following statements?

(i) Windows taskbar generally appears as a horizontal line at the bottom of the screen.
(ii) Windows 10 taskbar contains a search box which can be used to browse both the computer and the Web.

निम्नलिखित में से कौन-सा विकल्प निम्नलिखित कथनों की सत्यता का सर्वोत्तम वर्णन करता है?

(i) विंडोज़ टास्कबार (Windows taskbar) सामान्‍य रूप से स्क्रीन के निचले भाग पर एक क्षैतिज रेखा के रूप में दिखाई देता है।
(ii) विंडोज 10 टास्कबार (Windows 10 taskbar) में एक सर्च बॉक्स होता है जिसका उपयोग कंप्यूटर और वेब दोनों को ब्राउज़ करने के लिए किया जा सकता है

Options

A

(i) – True, (ii) – True

(i) – True, (ii) – True

B

(i) – False, (ii) – False

(i) – False, (ii) – False

C

(i) – True, (ii) – False

(i) – True, (ii) – False

D

(i) – False, (ii) – True

(i) – False, (ii) – True


Solution:

Correct Answer:

A

(i) – True, (ii) – True

(i) – सत्य, (ii) – सत्य


Question 31


Which of the following layers is NOT available in five layered internet architecture?

निम्नलिखित में से कौन-सी लेयर, फाइव-लेयर्ड इंटरनेट आर्किटेक्चर (five layered internet architecture) में उपलब्ध नहीं होता है?

Options

A

Session

Session

B

Transport

Transport

C

Application

Application

D

Data Link

Data Link


Solution:

Correct Answer:

A

Session

सेशन (Session)


Question 32


Which of the following is NOT a way to handle spam with reference to e-mail?

निम्नलिखित में से कौन-सा तथ्य, ई-मेल (e-mail) के संदर्भ में स्पैम (spam) को नियंत्रित करने का तरीका नहीं है?

Options

A

Think before you click any hyperlink

Think before you click any hyperlink

B

Give out or post your e-mail address publicly

Give out or post your e-mail address publicly

C

Do not reply to spam messages

Do not reply to spam messages

D

Download spam filtering tools and anti-virus software

Download spam filtering tools and anti-virus software


Solution:

Correct Answer:

B

Give out or post your e-mail address publicly

अपने ई-मेल (e-mail) पते को सार्वजनिक रूप से घोषित करना या पोस्ट करना


Question 33


A computer can directly understand which of the following languages?

एक कंप्यूटर निम्नलिखित में से किस भाषा (language) को सीधे समझ सकता है?

Options

A

High-level

High-level

B

Medium level

Medium level

C

Low-level

Low-level

D

Object level

Object level


Solution:

Correct Answer:

C

Low-level

निम्‍न-स्‍तर (Low-level)


Question 34


Which of the following is a computer-based application for the exchange of messages between users?

उपयोगकर्ताओं के बीच संदेशों के आदान-प्रदान के लिए निम्नलिखित में से कौन-सा कंप्यूटर आधारित एक एप्‍लीकेशन है?

Options

A

e-kyc

e-kyc

B

e-aadhar

e-aadhar

C

e-mail

e-mail

D

e-sharm

e-sharm


Solution:

Correct Answer:

C

e-mail

ई-मेल (e-mail)


Question 35


Which of the following is a world-wide network of computers, also known as network of networks?

निम्नलिखित में से कौन-सा टर्म, कंप्यूटरों का एक विश्वव्यापी नेटवर्क है, जिसे नेटवर्कों का नेटवर्क भी कहा जाता है?

Options

A

VLAN

VLAN

B

Internet

Internet

C

World Wide Web

World Wide Web

D

MAN

MAN


Solution:

Correct Answer:

B

Internet

इंटरनेट (Internet)


Question 36


Which of the following is NOT true with reference to searching for content on the internet?

इंटरनेट पर सामग्री को सर्च करने के संदर्भ में निम्नलिखित में से कौन-सा तथ्य सत्य नहीं है?

Options

A

Use specific keywords

Use specific keywords

B

Vary your search engine

Vary your search engine

C

Use quotation marks

Use quotation marks

D

Use complex search terms

Use complex search terms


Solution:

Correct Answer:

D

Use complex search terms

जटिल सर्च टर्म का उपयोग करना (Use complex search terms)


Question 37


IBM-360 series belongs to which of the following generations of computers?

IBM-360 सीरीज, कंप्यूटर की निम्नलिखित में से किस पीढ़ी से संबंधित है?

Options

A

Second

Second

B

Third

Third

C

Fourth

Fourth

D

First

First


Solution:

Correct Answer:

B

Third

तृतीय


Question 38


Which of the following in Gmail is the correct option to turn on the spell checking facility while composing e-mail message at any time?

किसी भी समय ई-मेल (e-mail) संदेश लिखते समय वर्तनी जाँच सुविधा को चालू करने के लिए जीमेल (Gmail) में निम्नलिखित में से कौन-सा सही विकल्प है?

Options

A

Settings > See all settings > General > Language

Settings > See all settings > General > Language

B

Settings > See all settings > General > Spelling

Settings > See all settings > General > Spelling

C

Settings > See all settings > Labels > Language

Settings > See all settings > Labels > Language

D

Settings > See all settings > Labels > Spelling

Settings > See all settings > Labels > Spelling


Solution:

Correct Answer:

B

Settings > See all settings > General > Spelling

सेटिंग्‍स > सी ऑल सेटिंग्‍स > जेनरल > स्‍पेलिंग
(Settings > See all settings > General > Spelling)


Question 39


Which of the following is the correct option to print a worksheet in MS-Excel 365?

MS-Excel 365 में वर्कशीट को प्रिंट करने का सही विकल्प निम्नलिखित में से कौन-सा है?

Options

A

Home -> Print

Home -> Print

B

File -> Print

File -> Print

C

Page Layout -> Print

Page Layout -> Print

D

View -> Print

View -> Print


Solution:

Correct Answer:

B

File -> Print

File -> Print


Question 40


Which of the following is true to replace specific characters of a cell in MS-Excel 365?

MS-Excel 365 में सेल के विशिष्ट अक्षर को प्रतिस्‍थापित करने के लिए निम्नलिखित में से कौन-सा तथ्य सही है?

Options

A

Click in the cell and then press Delete

Click in the cell and then press Delete

B

Click in the cell and then type the new characters

Click in the cell and then type the new characters

C

Click in the cell and then press Backspace

Click in the cell and then press Backspace

D

Select specific characters and then type the new characters

Select specific characters and then type the new characters


Solution:

Correct Answer:

D

Select specific characters and then type the new characters

विशिष्ट अक्षरों को सेलेक्ट करना और फिर नए अक्षरों को टाइप करना


Question 41


Which of the following is a great way to quickly navigate back to the websites we visit the most?

जिन वेबसाइटों पर हम सबसे अधिक जाते हैं, उन पर शीघ्रता से नेविगेट (navigate) करने के लिए निम्नलिखित में से कौन-सा विकल्‍प एक अच्‍छा तरीका है?

Options

A

Help

Help

B

Favorites

Favorites

C

Downloads

Downloads

D

History

History


Solution:

Correct Answer:

B

Favorites

फेवरिट्स (Favorites)


Question 42


Which of the following options is the correct sequence to print an entire web page in the Google Chrome browser?

निम्नलिखित में से कौन सा विकल्प गूगल क्रोम ब्राउज़र (Google Chromebrowser) में एक संपूर्ण वेब पेज को प्रिंट करने का सही क्रम है?

Options

A

Open the web page -> Press Ctrl + A -> Right click on the page and left click on ‘Print’

Open the web page -> Press Ctrl + A -> Right click on the page and left click on ‘Print’

B

Open the web page -> Click on Page Setup -> Click on the ‘Print’ icon

Open the web page -> Click on Page Setup -> Click on the ‘Print’ icon

C

Open the web page -> Press Ctrl + S -> Right click on the page and left click on ‘Print’

Open the web page -> Press Ctrl + S -> Right click on the page and left click on ‘Print’

D

Open the web page -> Click on Print Preview -> Click on the ‘Print’ icon

Open the web page -> Click on Print Preview -> Click on the ‘Print’ icon


Solution:

Correct Answer:

A

Open the web page -> Press Ctrl + A -> Right click on the page and left click on ‘Print’

वेब पेज खोलें -> Ctrl + A दबाएं -> पेज पर राइट क्लिक करें और ‘प्रिंट’ पर लेफ्ट क्लिक करें
(Open the web page -> Press Ctrl + A -> Right click on the page and left click on ‘Print’)


Question 43


Which of the following menus is used for adding clip art pictures to a presentation in MS-PowerPoint 365?

एमएस-पॉवरपॉइंट 365 में किसी प्रेजेंटेशन में क्लिप आर्ट चित्रों को डालने के लिए निम्नलिखित में से किस मेनू का उपयोग किया जाता है?

Options

A

Insert

Insert

B

Transitions

Transitions

C

Animation

Animation

D

Design

Design


Solution:

Correct Answer:

A

Insert

इन्सर्ट (Insert)


Question 44


Which of the following options of the Set Up Slide Show is used to loop the slide show until the people watching press the Esc key in MS-PowerPoint 365?

एमएस-पॉवरपॉइंट 365 में, सेट अप स्लाइड शो (Set Up Slide Show) के निम्नलिखित में से किस विकल्प का उपयोग, स्लाइड शो को तब तक लूप करने के लिए किया जाता है, जब तक कि इसे देखने वाले लोग Esc कुंजी(key) न दबाएँ?

Options

A

Browsed at a kiosk (full screen)

Browsed at a kiosk (full screen)

B

Presented by an individual (full screen)

Presented by an individual (full screen)

C

Browsed by an individual (window)

Browsed by an individual (window)

D

Presented by a speaker (full screen)

Presented by a speaker (full screen)


Solution:

Correct Answer:

A

Browsed at a kiosk (full screen)

ब्राउज्ड एट ए कियोस्‍क (फुल स्क्रीन) (Browsed at a kiosk (full screen)


Question 45


निम्नलिखित में से कौन-सा विकलप, सरकार और लोगों के बीच सबसे कुशल और सफल माध्यमों में से एक है और IECT के द्वारा प्राप्त किया जाता है?

निम्नलिखित में से कौन-सा विकलप, सरकार और लोगों के बीच सबसे कुशल और सफल माध्यमों में से एक है और IECT के द्वारा प्राप्त किया जाता है?

Options

A

ई-मनी (e-money)

ई-मनी (e-money)

B

ई-गवर्नेंस (e-governance)

ई-गवर्नेंस (e-governance)

C

ई-बिजनेस (e-business)

ई-बिजनेस (e-business)

D

ई-ऑफिस (e-office)

ई-ऑफिस (e-office)


Solution:

Correct Answer:

B

ई-गवर्नेंस (e-governance)

ई-गवर्नेंस (e-governance)


Question 46


Which of the following keyboard shortcuts is used, if we want to enter data in the same cell but on a new line in MS-Excel 365?

यदि हम MS-Excel 365 में एक ही सेल में, लेकिन एक नई लाइन पर डेटा प्रविष्‍ट करना चाहते हैं, तो निम्नलिखित में से किस कीबोर्ड शॉर्टकट (keyboard shortcuts) का उपयोग किया जाएगा?

Options

A

Alt + Esc

Alt + Esc

B

Ctrl + Esc

Ctrl + Esc

C

Alt + Enter

Alt + Enter

D

Shift + Enter

Shift + Enter


Solution:

Correct Answer:

C

Alt + Enter

Alt + Enter


Question 47


Which of the following image represents an icon used for removing all formatting from selected text in MS-Word 365?

निम्नलिखित में से कौन-सी इमेज, MS-Word 365 में सेलेक्टेड टेक्स्ट से सभी फॉर्मेटिंग को हटाने के लिए उपयोग किए जाने वाले आइकन को दर्शाती है?

Options

A

B

C

D


Solution:

Correct Answer:

D


Question 48


Which of the following formatting options connects text to the top, middle, or bottom of a bounding box in MS-PowerPoint 365?

निम्नलिखित में से कौन-सा फॉरमेटिंग विकल्प एमएस-पॉवरपॉइंट 365 में टेक्स्ट को बाउंडिंग बॉक्स (bounding box) के टॉप, मिडल या बॉटम से जोड़ता है?

Options

A

Add or Remove Columns

Add or Remove Columns

B

Align Text

Align Text

C

Text Direction

Text Direction

D

Numbering or Bullets

Numbering or Bullets


Solution:

Correct Answer:

B

Align Text

एलाइन टेक्स्ट (Align Text)


Question 49


Page Borders option is available in which of the following tabs in MS-Word 365?

MS-Word 365 में निम्नलिखित में से किस टैब में पेज बॉर्डर्स (Page Borders) विकल्प उपलब्ध होते है?

Options

A

Design

Design

B

Home

Home

C

Developer

Developer

D

Review

Review


Solution:

Correct Answer:

A

Design

डिज़ाइन (Design)


Question 50


Which of the following printers creates characters by striking pins against an ink ribbon?

निम्नलिखित में से कौन-सा प्रिंटर, एक इंक-रिबन पर पिन मारकर अक्षरों को छापता है?

Options

A

LED

LED

B

Inkjet

Inkjet

C

Laser

Laser

D

Dot matrix

Dot matrix


Solution:

Correct Answer:

D

Dot matrix

डॉट मैट्रिक्स (Dot matrix)


Question 51


On 10 April 2022, who among the following inaugurated the annual Madhavpur Mela in Gujarat?

10 अप्रैल 2022 को गुजरात में वार्षिक माधवपुर मेले का उद्घाटन निम्नलिखित में से किसने किया?

Options

A

Narendra Modi

Narendra Modi

B

Ram Nath Kovind

Ram Nath Kovind

C

Venkaiah Naidu

Venkaiah Naidu

D

Amit Shah

Amit Shah


Solution:

Correct Answer:

B

Ram Nath Kovind

राम नाथ कोविंद


Question 52


Who among the following players has won the World Games Athlete of the Year award for 2021?

वर्ष 2021 का वर्ल्ड गेम्स एथलीट ऑफ द ईयर पुरस्कार विजेता खिलाड़ी, निम्नलिखित में से कौन है?

Options

A

Rupinder Pal Singh

Rupinder Pal Singh

B

Parattu Raveendran Sreejesh

Parattu Raveendran Sreejesh

C

Suraj Karkera

Suraj Karkera

D

Lalit Kumar Upadhyay

Lalit Kumar Upadhyay


Solution:

Correct Answer:

B

Parattu Raveendran Sreejesh

परत्तु रवींद्रन श्रीजेश


Question 53


In which country is the headquarters of the Asian Development Bank located?

एशियाई विकास बैंक (Asian Development Bank) का मुख्यालय किस देश में स्थित है?

Options

A

The Philippines

The Philippines

B

Thailand

Thailand

C

Vietnam

Vietnam

D

Singapore

Singapore


Solution:

Correct Answer:

A

The Philippines

फिलीपींस


Question 54


Which of the following Vedas consists of ten mandalas and 1028 hymns?

निम्नलिखित में से किस वेद में दस मंडल और 1028 सूक्त हैं?

Options

A

Atharva Veda

Atharva Veda

B

Yajur Veda

Yajur Veda

C

Sama Veda

Sama Veda

D

Rig Veda

Rig Veda


Solution:

Correct Answer:

D

Rig Veda

ऋग्वेद


Question 55


प्रधान मंत्री नरेंद्र मोदी ने ______ में स्मृति वन नामक भारत के प्रथम भूकंप स्मारक का उद्घाटन किया।

प्रधान मंत्री नरेंद्र मोदी ने ______ में स्मृति वन नामक भारत के प्रथम भूकंप स्मारक का उद्घाटन किया।

Options

A

सूरत

सूरत

B

वाराणसी

वाराणसी

C

भुज

भुज

D

अहमदाबाद

अहमदाबाद


Solution:

Correct Answer:

C

भुज

भुज


Question 56


निम्नलिखित में से भारत के संविधान का कौन सा भाग राजभाषा से संबंधित है?

निम्नलिखित में से भारत के संविधान का कौन सा भाग राजभाषा से संबंधित है?

Options

A

भाग XIV

भाग XIV

B

भाग XVI

भाग XVI

C

भाग XV

भाग XV

D

भाग XVII

भाग XVII


Solution:

Correct Answer:

D

भाग XVII

भाग XVII


Question 57


Which Article of the Constitution of India is related to the appointment of ad hoc judges?

निम्नलिखित में से भारत के संविधान का कौन सा अनुच्छेद तदर्थ (ad hoc) न्यायाधीशों की नियुक्ति से संबंधित है?

Options

A

Article 152

Article 152

B

Article 130

Article 130

C

Article 143

Article 143

D

Article 127

Article 127


Solution:

Correct Answer:

D

Article 127

अनुच्छेद 127


Question 58


The 13th edition of the India-US joint special forces exercise, ‘Vajra Prahar 2022’ concluded at:

भारत – यूएस (US) विशेष सैन्य बल अभ्यास का 13वां संस्करण ‘वज्र प्रहार, 2022’______ में संपन्‍न हुआ।

Options

A

Pune

Pune

B

Bakloh

Bakloh

C

Jodhpur

Jodhpur

D

Chandipur

Chandipur


Solution:

Correct Answer:

B

Bakloh

बकलोह


Question 59


In which year was Mahatma Gandhi first imprisoned in Sabarmati central jail?

महात्मा गांधी को पहली बार साबरमती सेंट्रल जेल में किस वर्ष कैद किया गया था?

Options

A

1922

1922

B

1920

1920

C

1930

1930

D

1924

1924


Solution:

Correct Answer:

A

1922

1922


Question 60


The book ‘Development as Freedom’ is written by ______,an Indian economist.

'डेवलपमेंट ऐज़ फ्रीडम (Development as Freedom)' पुस्तक के लेखक भारतीय अर्थशास्त्री______ हैं।

Options

A

Montek Singh Ahluwalia

Montek Singh Ahluwalia

B

Amartya Sen

Amartya Sen

C

Amiya Kumar Bagchi

Amiya Kumar Bagchi

D

Amit Mitra

Amit Mitra


Solution:

Correct Answer:

B

Amartya Sen

अमर्त्य सेन


Question 61


The Securities and Exchange Board of India was constituted as a non-statutory body on ______ through a resolution of the Government of India.

भारतीय प्रतिभूति और विनिमय बोर्ड का गठन भारत सरकार के एक प्रस्ताव के माध्यम से ______ को एक गैर-सांविधिक निकाय के रूप में किया गया था।

Options

A

15 June 1988

15 June 1988

B

12 April 1988

12 April 1988

C

23 December 1990

23 December 1990

D

30 January 1992

30 January 1992


Solution:

Correct Answer:

B

12 April 1988

12 अप्रैल 1988


Question 62


In August 2022, with which institution did the Athletics Federation of India (AFI) announce its collaboration in order to support Indian women athletes?

अगस्त 2022 में, भारतीय एथलेटिक्स महासंघ (AFI) ने भारतीय महिला एथलीटों का समर्थन करने के लिए किस संस्थान के साथ अपनी सहभाग‍िता की घोषणा की?

Options

A

JP Morgan Chase & Co.

JP Morgan Chase & Co.

B

The Hongkong and Shanghai Banking Corporation Limited

The Hongkong and Shanghai Banking Corporation Limited

C

Central Bank of the Russian Federation 

Central Bank of the Russian Federation 

D

Sumitomo Mitsui Banking Corporation Limited

Sumitomo Mitsui Banking Corporation Limited


Solution:

Correct Answer:

B

The Hongkong and Shanghai Banking Corporation Limited

द हांगकांग एंड शंघाई बैंकिंग कॉर्पोरेशन लिमिटेड (The Hongkong and Shanghai Banking Corporation Limited)


Question 63


The situation wherein prices of goods generally increase and the purchasing value of money falls in an economy is called:

वह स्थिति जिसमें किसी अर्थव्यवस्था में सामान की कीमतें आम तौर पर बढ़ जाती हैं और मुद्रा का क्रय मूल्य गिर जाता है,______ कहलाती है।

Options

A

inflation

inflation

B

disinflation

disinflation

C

depreciation

depreciation

D

stagflation

stagflation


Solution:

Correct Answer:

A

inflation

मुद्रास्फीति


Question 64


In which of the following countries is Narita International Airport located?

नर‍िता (Narita) अंतर्राष्ट्रीय हवाई अड्डा निम्नलिखित में से किस देश मेंस्थित है?

Options

A

China

China

B

Japan

Japan

C

Bangladesh

Bangladesh

D

Indonesia

Indonesia


Solution:

Correct Answer:

B

Japan

जापान


Question 65


Which of the following sites of the Indus valley civilization was excavated by Amarendra Nath from 1997 to 2000?

अमरेन्द्र नाथ ने 1997 से 2000 तक सिंधु घाटी सभ्यता के निम्नलिखित में से किस स्थल की खुदाई की थी?

Options

A

Mohenjo Daro

Mohenjo Daro

B

Rakhigarhi

Rakhigarhi

C

Kalibangan

Kalibangan

D

Lothal

Lothal


Solution:

Correct Answer:

B

Rakhigarhi

राखीगढ़ी


Question 66


Which route enhanced the connectivity to Assam and West Bengal from Bangladesh?

निम्नलिखित में से किस मार्ग ने बांग्लादेश से असम और पश्चिम बंगाल की कनेक्टिविटी को बढ़ाया?

Options

A

Jogbani-Biratnagar route

Jogbani-Biratnagar route

B

Imphal-Moreh route

Imphal-Moreh route

C

Haldibari-Chilahati route

Haldibari-Chilahati route

D

Jodhpur and Munabao

Jodhpur and Munabao


Solution:

Correct Answer:

C

Haldibari-Chilahati route

हल्दीबाड़ी-चिल्हाटी मार्ग


Question 67


What were the targeted and achieved growth rates of the First Five-Year Plan, respectively?

प्रथम पंचवर्षीय योजना की क्रमशः लक्षित और प्राप्त विकास दर क्या थी?

Options

A

2.4 % and 4.1%

2.4 % and 4.1%

B

2.1% and 3.6%

2.1% and 3.6%

C

4.1% and 2.9%

4.1% and 2.9%

D

3.6% and 2.4%

3.6% and 2.4%


Solution:

Correct Answer:

B

2.1% and 3.6%

2.1% और 3.6%


Question 68


भारतीय संविधान के निम्नलिखित में से कौन से संविधान संशोधन अधिनियम में हिंदी भाषा में संविधान के एक आधिकारिक पाठ के लिए प्रावधान किया गया और यह संविधान के हिंदी संस्करण को समान कानूनी वैधता प्रदान करता है?

भारतीय संविधान के निम्नलिखित में से कौन से संविधान संशोधन अधिनियम में हिंदी भाषा में संविधान के एक आधिकारिक पाठ के लिए प्रावधान किया गया और यह संविधान के हिंदी संस्करण को समान कानूनी वैधता प्रदान करता है?

Options

A

संविधान (पैंसठवां संशोधन) अधिनियम, 1990

संविधान (पैंसठवां संशोधन) अधिनियम, 1990

B

संविधान (उनहत्तरवां संशोधन) अधिनियम, 1991

संविधान (उनहत्तरवां संशोधन) अधिनियम, 1991

C

संविधान (अठ्ठावनवां संशोधन) अधिनियम, 1987

संविधान (अठ्ठावनवां संशोधन) अधिनियम, 1987

D

संविधान (इकहत्तरवां संशोधन) अधिनियम, 1992

संविधान (इकहत्तरवां संशोधन) अधिनियम, 1992


Solution:

Correct Answer:

C

संविधान (अठ्ठावनवां संशोधन) अधिनियम, 1987

संविधान (अठ्ठावनवां संशोधन) अधिनियम, 1987


Question 69


Who among the following was the Mughal emperor when the revolt in favour of Mirza Hakim occurred in1579–80?

निम्नलिखित में से क‍िस मुगल बादशाह के शासन काल के दौरान 1579-80 में मिर्जा हकीम के पक्ष में विद्रोह हुआ था?

Options

A

Babur

Babur

B

Akbar

Akbar

C

Shah Jahan

Shah Jahan

D

Humayun

Humayun


Solution:

Correct Answer:

B

Akbar

अकबर


Question 70


In June 2022, in which of the following states was the National Museum of Customs and GST, called ‘Dharohar’, inaugurated?

जून 2022 में, निम्नलिखित में से किस राज्य में 'धरोहर' नामक राष्ट्रीय सीमा शुल्क और जीएसटी (GST) संग्रहालय का उद्घाटन किया गया था?

Options

A

Assam

Assam

B

Kerala

Kerala

C

Goa

Goa

D

Manipur

Manipur


Solution:

Correct Answer:

C

Goa

गोवा


Question 71


Which of the following countries is NOT covered in the Barak Basin?

निम्नलिखित में से कौन सा देश बराक बेसिन (Barak Basin) में शामिल नहीं है?

Options

A

Myanmar

Myanmar

B

Nepal

Nepal

C

Bangladesh

Bangladesh

D

India

India


Solution:

Correct Answer:

B

Nepal

नेपाल


Question 72


In the election of the President of India, the value of each MLA’s vote is determined by dividing the population of the state by the number of MLAs in its Legislative Assembly and further dividing the quotient achieved by:

भारत के राष्ट्रपति के चुनाव में, प्रत्येक विधायक के वोट का मान, राज्य की जनसंख्या को उसकी विधान सभा में विधायकों की संख्या से विभाजित करके और इससे प्राप्त भागफल को ___________ से विभाजित करके निर्धारित किया जाता है।

Options

A

100000

100000

B

100

100

C

1000

1000

D

10000

10000


Solution:

Correct Answer:

C

1000

1000


Question 73


From which dynasty was Rajaraja I, who built the Brihadishvara Temple at Thanjavur?

तंजावुर में बृहदीश्वर मंदिर का निर्माण करने वाले राजाराज प्रथम (Rajaraja I) किस वंश से संबंध रखते थे?

Options

A

Pallava Dynasty

Pallava Dynasty

B

Pandya Dynasty

Pandya Dynasty

C

Chola Dynasty

Chola Dynasty

D

Shaishunaga Dynasty

Shaishunaga Dynasty


Solution:

Correct Answer:

C

Chola Dynasty

चोल वंश


Question 74


In which year did Akbar issue gold coins called ‘Ilahi coins’ to propagate his idea of a new religion called ‘Din-i-Illahi’?

अकबर ने किस वर्ष में 'दीन-ए-इलाही' नामक एक नए धर्म के अपने विचार का प्रचार करने के लिए 'इलाही सिक्के' नामक सोने के सिक्के जारी किए थे?

Options

A

1574

1574

B

1569

1569

C

1579

1579

D

1567

1567


Solution:

Correct Answer:

C

1579

1579


Question 75


The reservation for the Anglo-Indian community was extended till the year ______ through the Constitution (Ninety-fifth Amendment) Act, 2009.

संविधान (पचानवेवां संशोधन) अधिनियम, 2009 के माध्यम से एंग्लो-इंडियन समुदाय के लिए आरक्षण को वर्ष ______ तक बढ़ा दिया गया था।

Options

A

2015

2015

B

2020

2020

C

2030

2030

D

2025

2025


Solution:

Correct Answer:

B

2020

2020


Question 76


If × stands for −, ÷ stands for +, + stands for ÷, and − stands for ×, then which of the following equations is correct?  

यदि × का अर्थ − है, ÷ का अर्थ + है, + का अर्थ ÷ है, और − का अर्थ × है, तो निम्नलिखित में से कौन सा समीकरण सही है?

Options

A

76 ÷ 9 + 3 × 2 − 4 = 83

76 ÷ 9 + 3 × 2 − 4 = 83

B

46 ÷ 18 + 9 × 8 − 9 = 23

46 ÷ 18 + 9 × 8 − 9 = 23

C

36 ÷ 6 + 3 × 4 − 7 = 10

36 ÷ 6 + 3 × 4 − 7 = 10

D

35 ÷ 7 + 6 × 6 − 8 = 83

35 ÷ 7 + 6 × 6 − 8 = 83


Solution:

Correct Answer:

C

36 ÷ 6 + 3 × 4 − 7 = 10

36 ÷ 6 + 3 × 4 − 7 = 10


Question 77


Select the option that is related to the third number in the same way as the second number is related to the first number.

37 : 55.5 :: 29 : ?

उस विकल्प का चयन करें जो तीसरी संख्या से उसी प्रकार संबंधित है जैसे दूसरी संख्या पहली संख्या से संबंधित है।

37 : 55.5 :: 29 : ?

Options

A

45.5

45.5

B

41.5

41.5

C

46.5

46.5

D

43.5

43.5


Solution:

Correct Answer:

D

43.5

43.5


Question 78


In a certain code language, ‘MONK’ is coded as ‘212’. How will ‘DOG’ be coded in that language?

एक निश्चित कूट भाषा में, 'MONK' को '212' के रूप में कूटबद्ध किया जाता है। उसी भाषा में 'DOG' को किस प्रकार कूटबद्ध किया जाएगा?

Options

A

34

34

B

67

67

C

78

78

D

112

112


Solution:

Correct Answer:

B

67

67


Question 79


Select the number from among the given options that can replace the question mark (?) in the following series.

1695, 1598, 1521, 1464, ?

दिए गए विकल्पों में से वह संख्या चुनिए जो निम्नलिखित श्रृंखला में प्रश्नवाचक चिन्ह (?) को प्रतिस्थापित कर सकती है।

1695, 1598, 1521, 1464, ?

Options

A

1414

1414

B

1489

1489

C

1427

1427

D

1437

1437


Solution:

Correct Answer:

C

1427

1427


Question 80


In a certain code language, ‘HUSBAND’ is written as ‘ANDBHUS’. How will ‘KITCHEN’ be written in that language?

एक निश्चित कूट भाषा में, 'HUSBAND' को 'ANDBHUS' लिखा जाता है। उसी भाषा में 'KITCHEN' को कैसे लिखा जाएगा?

Options

A

HENCKTI

HENCKTI

B

HENDKIT

HENDKIT

C

HEMCKIT

HEMCKIT

D

HENCKIT

HENCKIT


Solution:

Correct Answer:

D

HENCKIT

HENCKIT


Question 81


उस विकल्प का चयन करें जो पांचवीं संख्या से उसी प्रकार संबंधित है जैसे दूसरी संख्या पहली संख्या से संबंधित है और चौथी संख्या तीसरी संख्या से संबंधित है।

15 : 90 :: 29 : 319 :: 34 : ?

उस विकल्प का चयन करें जो पांचवीं संख्या से उसी प्रकार संबंधित है जैसे दूसरी संख्या पहली संख्या से संबंधित है और चौथी संख्या तीसरी संख्या से संबंधित है।

15 : 90 :: 29 : 319 :: 34 : ?

Options

A

238

238

B

364

364

C

120

120

D

302

302


Solution:

Correct Answer:

A

238

238


Question 82


Select the alphanumeric-cluster from among the given options that can replace the question mark (?) in the following series.

45C, 54G, 48L, 57R, ?

दिए गए विकल्पों में से उस अक्षरांकीय-समूह का चयन करें जो निम्नलिखित श्रृंखला में प्रश्न चिह्न (?) को प्रतिस्थापित कर सकता है।

45C, 54G, 48L, 57R, ?

Options

A

66X

66X

B

51Y

51Y

C

62X

62X

D

53Z

53Z


Solution:

Correct Answer:

B

51Y

51Y


Question 83


यदि नीचे दिए गए अक्षरों को अंग्रेजी वर्णमाला श्रृंखला में बाएं से दाएं उनके स्थान के आरोही क्रम में व्यवस्थित किया जाता है, तो बाएं से पहले, पांचवे, सातवें, बारहवें, सोलहवें, अट्ठारहवें और उन्‍नीसवें का उपयोग करके दिए गए विकल्पों में से कौन सा अर्थपूर्ण शब्द बनाया जा सकता है? सभी अक्षरों का उपयोग किया जाना चाहिए और प्रत्येक अक्षर का उपयोग केवल एक बार किया जाना है।
MUBYIJRLGDOPCVZQNAHK

यदि नीचे दिए गए अक्षरों को अंग्रेजी वर्णमाला श्रृंखला में बाएं से दाएं उनके स्थान के आरोही क्रम में व्यवस्थित किया जाता है, तो बाएं से पहले, पांचवे, सातवें, बारहवें, सोलहवें, अट्ठारहवें और उन्‍नीसवें का उपयोग करके दिए गए विकल्पों में से कौन सा अर्थपूर्ण शब्द बनाया जा सकता है? सभी अक्षरों का उपयोग किया जाना चाहिए और प्रत्येक अक्षर का उपयोग केवल एक बार किया जाना है।
MUBYIJRLGDOPCVZQNAHK

Options

A

DOING

DOING

B

BOILING

BOILING

C

VARYING

VARYING

D

RAINING

RAINING


Solution:

Correct Answer:

C

VARYING

VARYING


Question 84


Four letter-clusters have been given, out of which three are alike in some manner and one is different. Select the letter-cluster that is different.

चार अक्षर-समूह दिए गए हैं, जिनमें से तीन किसी न किसी रूप में एकसमान हैं और एक असंगत है। उस असंगत अक्षर-समूह का चयन करें।

Options

A

AGM

AGM

B

INS

INS

C

ELS

ELS

D

DHM

DHM


Solution:

Correct Answer:

D

DHM

DHM


Question 85


Select the answer figure that is related to problem figure C in the same way as problem figure B is related to problem figure A.


उस उत्तर आकृति का चयन करें जो प्रश्‍न आकृति C से उसी प्रकार संबंधित है जिस प्रकार प्रश्‍न आकृति B प्रश्‍न आकृति A से संबंधित है।


Options

A

1

1

B

3

3

C

2

2

D

4

4


Solution:

Correct Answer:

C

2

2


Question 86


In a certain code language,
‘A # B’ means ‘A is the mother of B’,
‘A % B’ means ‘A is the father of B’,
‘A & B’ means ‘A is the son of B’,
‘A @ B’ means ‘A is the brother of B’.
If ‘M % N & P # Q @ T’, how is M related to T?

एक निश्चित कूट भाषा में,
'A # B' का अर्थ है कि 'A, B की मां है',
'A% B' का अर्थ है कि 'A, B का पिता है',
'A & B' का अर्थ है कि 'A, B का पुत्र है',
'A @ B' का अर्थ है कि 'A, B का भाई है'।
यदि 'M% N & P # Q @ T' है, तो M का T से क्‍या रिश्‍ता है?

Options

A

Father

Father

B

Husband

Husband

C

Brother

Brother

D

Son

Son


Solution:

Correct Answer:

A

Father

पिता


Question 87


Select the option that is related to the third term in the same way as the second term is related to the first term.

SORROW : ROWROS :: DEGREE : ?

उस विकल्प का चयन करें जो तीसरे पद से उसी प्रकार संबंधित है जैसे दूसरा पद पहले पद से संबंधित है।

SORROW : ROWROS :: DEGREE : ?

Options

A

REEGED

REEGED

B

REEFED

REEFED

C

REEEGD

REEEGD

D

REEGDE

REEGDE


Solution:

Correct Answer:

A

REEGED

REEGED


Question 88


Four figures have been given, out of which three are alike in some manner and one is different. Select the figure that is different.

चार आकृति दी गई है, जिनमें से तीन किसी न किसी तरीके से एक समान हैं और एक असंगत है। असंगत आकृति का चयन करें।

Options

A

B

C

D


Solution:

Correct Answer:

A


Question 89


In a certain code language, ‘FOOT’ is coded as ‘56’. How will ‘COLD’ be coded in that language?

एक निश्चित कूट भाषा में, 'FOOT' को '56' के रूप में कूटबद्ध किया जाता है। उसी भाषा में 'COLD' को किस प्रकार कूटबद्ध किया जाएगा?

Options

A

37

37

B

34

34

C

43

43

D

41

41


Solution:

Correct Answer:

B

34

34


Question 90


Select the alphanumeric-cluster from among the given options that can replace the question mark (?) in the following series.

A121O, F169T, K225Y, P289D, ?

दिए गए विकल्पों में से उस अक्षरांकीय-समूह का चयन करें जो निम्नलिखित श्रृंखला में प्रश्न चिह्न (?) को प्रतिस्थापित कर सकता है।

A121O, F169T, K225Y, P289D, ?

Options

A

U316I

U316I

B

V361I

V361I

C

U361I

U361I

D

U361J

U361J


Solution:

Correct Answer:

C

U361I

U361I


Question 91


Select the option that is related to the third term in the same way as the second term is related to the first term.

SIRIUS : TVJSJT :: FELINE : ?

उस विकल्प का चयन करें जो तीसरे पद से उसी प्रकार संबंधित है जैसे दूसरा पद पहले पद से संबंधित है।

SIRIUS : TVJSJT :: FELINE : ?

Options

A

FOJOFG

FOJOFG

B

GEMJOF

GEMJOF

C

GEMIOF

GEMIOF

D

FOJMFG

FOJMFG


Solution:

Correct Answer:

D

FOJMFG

FOJMFG


Question 92


Mana is the son of Devika. Dev is the father of Hina, while Gul is the sister of Mana. If Gul is the sister of Hina, how is Dev related to Devika?

मन, देविका का पुत्र है। देव, हिना का पिता है, जबकि गुल, मन की बहन है। यदि गुल, हिना की बहन है, तो देव का देविका से क्‍या रिश्‍ता है?

Options

A

Son

Son

B

Brother

Brother

C

Father

Father

D

Husband

Husband


Solution:

Correct Answer:

D

Husband

पति


Question 93


In a certain code language, ‘CHIEF’ is written as ‘EJKGH’. How will ‘BAKED’ be written in that language?

एक निश्चित कूट भाषा में, 'CHIEF' को 'EJKGH' लिखा जाता है। उसी भाषा में 'BAKED' को कैसे लिखा जाएगा?

Options

A

DCMGF

DCMGF

B

DCNGF

DCNGF

C

DCMGE

DCMGE

D

DDMGF

DDMGF


Solution:

Correct Answer:

A

DCMGF

DCMGF


Question 94


Select the answer figure that is related to problem figure C in the same way as problem figure B is related to problem figure A.


उस उत्तर आकृति का चयन करें जो प्रश्‍न आकृति C से उसी प्रकार संबंधित है जिस प्रकार प्रश्‍न आकृति B प्रश्‍न आकृति A से संबंधित है।


Options

A

2

2

B

1

1

C

3

3

D

4

4


Solution:

Correct Answer:

C

3

3


Question 95


In a certain code language, ‘RKSTPN’ is coded as ‘131178169’. How will ‘NLTRHP’ be coded in that language?

एक निश्चित कूट भाषा में, 'RKSTPN' को '131178169' के रूप में कूटबद्ध किया गया है। उसी भाषा में 'NLTRHP' को किस प्रकार कूटबद्ध किया जाएगा?

Options

A

14122018816

14122018816

B

1315791911

1315791911

C

1119971513

1119971513

D

11199201513

11199201513


Solution:

Correct Answer:

C

1119971513

1119971513


Question 96


Select the alphanumeric-cluster from among the given options that can replace the question mark (?) in the following series.

12P, 18T, 25X, 33B, ?

दिए गए विकल्पों में से उस अक्षरांकीय-समूह का चयन करें जो निम्नलिखित श्रृंखला में प्रश्न चिह्न (?) को प्रतिस्थापित कर सकता है।

12P, 18T, 25X, 33B, ?

Options

A

39D

39D

B

42F

42F

C

41F

41F

D

42E

42E


Solution:

Correct Answer:

B

42F

42F


Question 97


ABCDEFGHIJKLMNOPQRSTUVWXYZ
यदि पहले, तीसरे, पांचवे, छठे, बारहवें, अट्ठारहवें और इक्‍कीसवें अक्षरों को एक अर्थपूर्ण शब्द बनाने के लिए चुना जाता है, तो मध्य स्थान पर कौन सा अक्षर आएगा? सभी चुने हुए अक्षरों का उपयोग किया जाना चाहिए और प्रत्येक अक्षर का उपयोग केवल एक बार किया जाना है।

ABCDEFGHIJKLMNOPQRSTUVWXYZ
यदि पहले, तीसरे, पांचवे, छठे, बारहवें, अट्ठारहवें और इक्‍कीसवें अक्षरों को एक अर्थपूर्ण शब्द बनाने के लिए चुना जाता है, तो मध्य स्थान पर कौन सा अक्षर आएगा? सभी चुने हुए अक्षरों का उपयोग किया जाना चाहिए और प्रत्येक अक्षर का उपयोग केवल एक बार किया जाना है।

Options

A

L

L

B

A

A

C

E

E

D

R

R


Solution:

Correct Answer:

D

R

R


Question 98


उस विकल्प का चयन करें जो तीसरे शब्द से उसी प्रकार संबंधित है जैसे दूसरा शब्द पहले शब्द से संबंधित है।

भोजन : बावर्ची :: शिकार : ?

उस विकल्प का चयन करें जो तीसरे शब्द से उसी प्रकार संबंधित है जैसे दूसरा शब्द पहले शब्द से संबंधित है।

भोजन : बावर्ची :: शिकार : ?

Options

A

शिकारी

शिकारी

B

दर्जी

दर्जी

C

कवि

कवि

D

न्यायाधीश

न्यायाधीश


Solution:

Correct Answer:

A

शिकारी

शिकारी


Question 99


In a certain code language,
‘A # B’ means ‘A is the mother of B’,
‘A % B’ means ‘A is the daughter of B’,
‘A & B’ means ‘A is the father of B’,
‘A @ B’ means ‘A is the brother of B’.
If ‘T @ U & R % S # G’, how is T related to G?

एक निश्चित कूट भाषा में,
'A # B' का अर्थ है कि ' A, B की मां है',
'A % B' का अर्थ है कि 'A, B की पुत्री है',
'A & B' का अर्थ है कि 'A, B का पिता है',
'A @ B' का अर्थ है कि 'A, B का भाई है'।
यदि 'T @ U & R % S # G' है, तो T का G से क्‍या रिश्‍ता है?

Options

A

Maternal uncle

Maternal uncle

B

Brother

Brother

C

Father

Father

D

Paternal uncle

Paternal uncle


Solution:

Correct Answer:

D

Paternal uncle

चाचा/फूफा


Question 100


Select the correct combination of mathematical signs that can sequentially replace the * signs and balance the following equation.

(17 * 4) * 225 * 15 * 83

गणितीय चिन्हों के उस सही संयोजन का चयन करें जो * चिन्हों को क्रमिक रूप से प्रतिस्थापित करके निम्नलिखित समीकरण को संतुलित कर सकता है।

(17 * 4) * 225 * 15 * 83

Options

A

÷, +, ×, =

÷, +, ×, =

B

−, ÷, +, =

−, ÷, +, =

C

×, +, ÷, =

×, +, ÷, =

D

+, ÷, ×, =

+, ÷, ×, =


Solution:

Correct Answer:

C

×, +, ÷, =

×, +, ÷, =


Question 101


Select the number from among the given options that can replace the question mark (?) in the following series.

4, 15, 41, 134, ?

दिए गए विकल्पों में से उस संख्या का चयन करें, जो निम्नलिखित श्रृंखला में प्रश्नवाचक चिह्न (?) को प्रतिस्थापित कर सकती है।

4, 15, 41, 134, ?

Options

A

457

457

B

754

754

C

745

745

D

547

547


Solution:

Correct Answer:

D

547

547


Question 102


Select the number from among the given options that can replace the question mark (?) in the following series.

2134, 2153, 2124, 2163, ?

दिए गए विकल्पों में से वह संख्या चुनिए जो निम्नलिखित श्रृंखला में प्रश्नवाचक चिन्ह (?) को प्रतिस्थापित कर सकती है।

2134, 2153, 2124, 2163, ?

Options

A

2114

2114

B

2202

2202

C

2212

2212

D

2143

2143


Solution:

Correct Answer:

A

2114

2114


Question 103


Select the option that is related to the third number in the same way as the second number is related to the first number.

64 : 2048 :: 84 : ?

उस विकल्प का चयन करें जो तीसरी संख्या से उसी प्रकार संबंधित है जैसे दूसरी संख्या पहली संख्या से संबंधित है।

64 : 2048 :: 84 : ?

Options

A

3488

3488

B

3528

3528

C

3378

3378

D

3890

3890


Solution:

Correct Answer:

B

3528

3528


Question 104


चार शब्द-त्रय दिए गए हैं, जिनमें से तीन किसी न किसी रूप में एकसमान हैं और एक असंगत है। उस असंगत शब्द-त्रय का चयन करें।

चार शब्द-त्रय दिए गए हैं, जिनमें से तीन किसी न किसी रूप में एकसमान हैं और एक असंगत है। उस असंगत शब्द-त्रय का चयन करें।

Options

A

गैराज-कार-किट्टेन (Garage-Car-Kitten)

गैराज-कार-किट्टेन (Garage-Car-Kitten)

B

व्हेल-डॉल्फ़िन-बिल्‍ली (Whale-Dolphin-Cat)

व्हेल-डॉल्फ़िन-बिल्‍ली (Whale-Dolphin-Cat)

C

मांद-घोंसला-बाड़ा (Den-Nest-Barn)

मांद-घोंसला-बाड़ा (Den-Nest-Barn)

D

दहाड़-म्याऊं-मिमियाना (Roar-Mew-Bleat)

दहाड़-म्याऊं-मिमियाना (Roar-Mew-Bleat)


Solution:

Correct Answer:

A

गैराज-कार-किट्टेन (Garage-Car-Kitten)

गैराज-कार-किट्टेन (Garage-Car-Kitten)


Question 105


चार शब्द दिए गए हैं, जिनमें से तीन किसी तरह से एकसमान हैं और एक असंगत है। उस असंगत शब्द का चयन करें।

चार शब्द दिए गए हैं, जिनमें से तीन किसी तरह से एकसमान हैं और एक असंगत है। उस असंगत शब्द का चयन करें।

Options

A

किलोग्राम

किलोग्राम

B

फुट

फुट

C

समय

समय

D

लीटर

लीटर


Solution:

Correct Answer:

C

समय

समय


Question 106


Four words have been given, out of which three are alike in some manner and one is different. Select the word that is different.

चार शब्द दिए गए हैं, जिनमें से तीन किसी तरह से एकसमान हैं और एक असंगत है। उस असंगत शब्द का चयन करें।

Options

A

Larva

Larva

B

Tadpole

Tadpole

C

Gang

Gang

D

Fawn

Fawn


Solution:

Correct Answer:

C

Gang

गैंग


Question 107


Select the number from among the given options that can replace the question mark (?) in the following series.

4532, 4316, 4191, 4127, ?

दिए गए विकल्पों में से वह संख्या चुनिए जो निम्नलिखित श्रृंखला में प्रश्नवाचक चिन्ह (?) को प्रतिस्थापित कर सकती है।

4532, 4316, 4191, 4127, ?

Options

A

4119

4119

B

4100

4100

C

4112

4112

D

4118

4118


Solution:

Correct Answer:

B

4100

4100


Question 108


Select the option that is related to the third word in the same way as the second word is related to the first word.

Kennel : Dog :: Aviary : ?

उस विकल्प का चयन करें जो तीसरे शब्द से उसी प्रकार संबंधित है जैसे दूसरा शब्द पहले शब्द से संबंधित है।

केनेल : कुत्ता :: एवियरी : ?

Options

A

Bird

Bird

B

Rider

Rider

C

Cattle

Cattle

D

Singer

Singer


Solution:

Correct Answer:

A

Bird

चिड़िया


Question 109


Four figures have been given, out of which three are alike in some manner and one is different. Select the figure that is different.


चार आकृतियां दी गई हैं, जिनमें से तीन किसी न किसी रूप में एकसमान हैं और एक असंगत है। उस असंगत आकृति का चयन करें।


Options

A

1

1

B

4

4

C

2

2

D

3

3


Solution:

Correct Answer:

A

1

1


Question 110


Select the number from among the given options that can replace the question mark (?) in the following series.

1654, 1671, 1705, 1756, ?

दिए गए विकल्पों में से वह संख्या चुनिए जो निम्नलिखित श्रृंखला में प्रश्नवाचक चिन्ह (?) को प्रतिस्थापित कर सकती है।

1654, 1671, 1705, 1756, ?

Options

A

1804

1804

B

1824

1824

C

1842

1842

D

1874

1874


Solution:

Correct Answer:

B

1824

1824


Question 111


उस शब्द का चयन करें जो दिए गए स्ट्रिंग में अक्षरों का उतनी ही बार उपयोग करके नहीं बनाया जा सकता है जितनी बार वे स्ट्रिंग में आए हैं।

HJWKSLPEUTNRAOCB

उस शब्द का चयन करें जो दिए गए स्ट्रिंग में अक्षरों का उतनी ही बार उपयोग करके नहीं बनाया जा सकता है जितनी बार वे स्ट्रिंग में आए हैं।

HJWKSLPEUTNRAOCB

Options

A

PENUTS

PENUTS

B

WEST

WEST

C

KNOW

KNOW

D

PETER

PETER


Solution:

Correct Answer:

D

PETER

PETER


Question 112


Four letter-clusters have been given, out of which three are alike in some manner and one is different. Select the letter-cluster that is different.

चार अक्षर-समूह दिए गए हैं, जिनमें से तीन किसी न किसी रूप में एकसमान हैं और एक असंगत है। उस असंगत अक्षर-समूह का चयन करें।

Options

A

GIKM

GIKM

B

OQSU

OQSU

C

BDFH

BDFH

D

PRSU

PRSU


Solution:

Correct Answer:

D

PRSU

PRSU


Question 113


Four figures have been given, out of which three are alike in some manner and one is different. Select the figure that is different.


चार आकृतियां दी गई हैं, जिनमें से तीन किसी न किसी रूप में एकसमान हैं और एक असंगत है। उस असंग आकृति का चयन करें।


Options

A

4

4

B

2

2

C

3

3

D

1

1


Solution:

Correct Answer:

C

3

3


Question 114


Four numbers have been given, out of which three are alike in some manner and one is different. Select the number that is different.

चार संख्‍याएं दी गई हैं, जिनमें से तीन किसी न किसी रूप में एकसमान हैं और एक असंगत है। उस असंगत संख्या का चयन करें।

Options

A

223

223

B

267

267

C

211

211

D

233

233


Solution:

Correct Answer:

B

267

267


Question 115


M and N are sisters, while P and Q are brothers. S is the son of P and the brother of N. How is M related to Q?

M और N बहनें हैं, जबकि P और Q भाई हैं। S, P का पुत्र है और N का भाई है। M का Q से क्‍या रिश्‍ता है?

Options

A

Daughter

Daughter

B

Wife

Wife

C

Brother’s daughter

Brother’s daughter

D

Brother’s son

Brother’s son


Solution:

Correct Answer:

C

Brother’s daughter

भतीजी


Question 116


Four numbers have been given, out of which three are alike in some manner and one is different. Select the number that is different.

चार संख्‍याएं दी गई हैं, जिनमें से तीन किसी न किसी रूप में एकसमान हैं और एक असंगत है। उस असंगत संख्या का चयन करें।

Options

A

168

168

B

224

224

C

362

362

D

288

288


Solution:

Correct Answer:

C

362

362


Question 117


In a certain code language,
‘A + B’ means ‘A is the daughter of B’,
‘A × B’ means ‘A is the father of B’,
‘A ÷ B’ means ‘A is the brother of B’.
If ‘M + N × P ÷ Q’, how is M related to Q?

एक निश्चित कूट भाषा में,
‘A + B’ का अर्थ है कि 'A, B की पुत्री है',
‘A × B’ का अर्थ है कि 'A, B का पिता है',
‘A ÷ B’ का अर्थ है कि 'A, B का भाई है'।
यदि 'M + N × P ÷ Q' है, तो M का Q से क्‍या रिश्‍ता है?

Options

A

Mother

Mother

B

Daughter

Daughter

C

Wife

Wife

D

Sister

Sister


Solution:

Correct Answer:

D

Sister

बहन


Question 118


Four numbers have been given, out of which three are alike in some manner and one is different. Select the number that is different.

चार संख्‍याएं दी गई हैं, जिनमें से तीन किसी न किसी रूप में एकसमान हैं और एक असंगत है। उस असंगत संख्या का चयन करें।

Options

A

119

119

B

51

51

C

85

85

D

105

105


Solution:

Correct Answer:

D

105

105


Question 119


If Q means ‘add to’, J means ‘multiply by’, T means ‘subtract from’, and K means ‘divide by’, then what will be the value of the following expression?

306 K 9 Q 37 J 8 T 47

यदि Q का अर्थ 'में जोड़ना' है, J का अर्थ 'से गुणा करना' है, T का अर्थ 'में से घटाना' है, और K का अर्थ 'से भाग देना' है, तो निम्नलिखित व्यंजक का मान क्या होगा?

306 K 9 Q 37 J 8 T 47

Options

A

283

283

B

289

289

C

301

301

D

317

317


Solution:

Correct Answer:

A

283

283


Question 120


Four figures have been given, out of which three are alike in some manner and one is different. Select the figure that is different.


चार आकृतियां दी गई हैं, जिनमें से तीन किसी न किसी रूप में एकसमान हैं और एक असंगत है। उस असंगत आकृति का चयन करें।



Options

A

4

4

B

3

3

C

1

1

D

2

2


Solution:

Correct Answer:

B

3

3


Question 121


निम्नलिखित में से कौन सा शब्द ‘अम्बर’ का पर्यायवाची नहीं है?

निम्नलिखित में से कौन सा शब्द ‘अम्बर’ का पर्यायवाची नहीं है?

Options

A

कस्तूरी

कस्तूरी

B

पायल

पायल

C

व्योम

व्योम

D

परिधान

परिधान


Solution:

Correct Answer:

B

पायल

पायल


Question 122


निम्नलिखित में से किस मुहावरे का अर्थ ‘अपना काम स्वयं करना’ है?

निम्नलिखित में से किस मुहावरे का अर्थ ‘अपना काम स्वयं करना’ है?

Options

A

अपनी करनी पार उतरनी

अपनी करनी पार उतरनी

B

अपनी पगड़ी अपने हाथ

अपनी पगड़ी अपने हाथ

C

अपना हाथ जगन्नाथ

अपना हाथ जगन्नाथ

D

अपनी-अपनी ढपली, अपना-अपना राग

अपनी-अपनी ढपली, अपना-अपना राग


Solution:

Correct Answer:

C

अपना हाथ जगन्नाथ

अपना हाथ जगन्नाथ


Question 123


निम्नलिखित वाक्य के अशुद्ध भाग को चिह्नित कीजिए –
‘आप सभी को अच्छी तरह से यह समझ लेना चाहिए कि तुष्टिकरण करने की यह नीति हमारे व्यवसाय के हित में नहीं है|’

निम्नलिखित वाक्य के अशुद्ध भाग को चिह्नित कीजिए –
‘आप सभी को अच्छी तरह से यह समझ लेना चाहिए कि तुष्टिकरण करने की यह नीति हमारे व्यवसाय के हित में नहीं है|’

Options

A

तुष्टिकरण करने की यह नीति

तुष्टिकरण करने की यह नीति

B

यह समझ लेना चाहिए कि

यह समझ लेना चाहिए कि

C

आप सभी को अच्छी तरह से

आप सभी को अच्छी तरह से

D

हमारे व्यवसाय के हित में नहीं है

हमारे व्यवसाय के हित में नहीं है


Solution:

Correct Answer:

A

तुष्टिकरण करने की यह नीति

तुष्टिकरण करने की यह नीति


Question 124


निम्नलिखित में से किस लोकोक्ति का अर्थ ‘सख्ती करने से सब काबू में आते हैं’ है?

निम्नलिखित में से किस लोकोक्ति का अर्थ ‘सख्ती करने से सब काबू में आते हैं’ है?

Options

A

डंडा सबका पीर

डंडा सबका पीर

B

जिसकी लाठी उसकी भैस

जिसकी लाठी उसकी भैस

C

ज़बरा मारे रोने न दे

ज़बरा मारे रोने न दे

D

जिसके हाथ डोई, उसका सब कोई

जिसके हाथ डोई, उसका सब कोई


Solution:

Correct Answer:

A

डंडा सबका पीर

डंडा सबका पीर


Question 125


‘लूट का मूसल भी बहुत’ की समानार्थी लोकोक्ति का चयन करें-

‘लूट का मूसल भी बहुत’ की समानार्थी लोकोक्ति का चयन करें-

Options

A

ले दे आटा कठौती में

ले दे आटा कठौती में

B

लूट कोयलों की मार बर्छी की

लूट कोयलों की मार बर्छी की

C

लूट में चरखा नफ़ा

लूट में चरखा नफ़ा

D

लूट लाया, कूट खाया

लूट लाया, कूट खाया


Solution:

Correct Answer:

C

लूट में चरखा नफ़ा

लूट में चरखा नफ़ा


Question 126


‘कंदर्प’ के विलोम शब्द का चयन करें-

‘कंदर्प’ के विलोम शब्द का चयन करें-

Options

A

बेकद्री

बेकद्री

B

दर्प

दर्प

C

मजबूत

मजबूत

D

मदनप्रिया

मदनप्रिया


Solution:

Correct Answer:

D

मदनप्रिया

मदनप्रिया


Question 127


निम्नलिखित में से अशुद्ध विलोम-युग्म की पहचान करें-

निम्नलिखित में से अशुद्ध विलोम-युग्म की पहचान करें-

Options

A

रूढ़िलक्षणा – प्रयोजनवती

रूढ़िलक्षणा – प्रयोजनवती

B

रूक्ष – स्निग्ध

रूक्ष – स्निग्ध

C

अधूत – अवधूत

अधूत – अवधूत

D

प्रेषक – प्रापक

प्रेषक – प्रापक


Solution:

Correct Answer:

C

अधूत – अवधूत

अधूत – अवधूत


Question 128


निम्नलिखित वाक्यों में से शुद्ध वाक्य का चयन कीजिये-

निम्नलिखित वाक्यों में से शुद्ध वाक्य का चयन कीजिये-

Options

A

राम और सीता चौदह वर्षों के बाद अयोध्या आयीं|

राम और सीता चौदह वर्षों के बाद अयोध्या आयीं|

B

वे सब प्रकार के दोषों से मुक्त हैं|

वे सब प्रकार के दोषों से मुक्त हैं|

C

आज तो दही खट्टी है |

आज तो दही खट्टी है |

D

नेता की मृत्यु का समाचार से सम्पूर्ण देश भर में निराशा छा गयी |

नेता की मृत्यु का समाचार से सम्पूर्ण देश भर में निराशा छा गयी |


Solution:

Correct Answer:

B

वे सब प्रकार के दोषों से मुक्त हैं|

वे सब प्रकार के दोषों से मुक्त हैं|


Question 129


निम्नलिखित में से कौन सा विकल्प शुद्ध तद्भव- तत्सम युग्म है?

निम्नलिखित में से कौन सा विकल्प शुद्ध तद्भव- तत्सम युग्म है?

Options

A

सोंठ – शुष्ठि

सोंठ – शुष्ठि

B

आँख – चक्षु

आँख – चक्षु

C

घृत – घी

घृत – घी

D

हाथ – हस्ती

हाथ – हस्ती


Solution:

Correct Answer:

A

सोंठ – शुष्ठि

सोंठ – शुष्ठि


Question 130


‘चुल्लू-चुल्लू साधना’ मुहावरे के अर्थ का चयन कीजिये-

‘चुल्लू-चुल्लू साधना’ मुहावरे के अर्थ का चयन कीजिये-

Options

A

ज्यादा शरमा जाना

ज्यादा शरमा जाना

B

थोड़ा-थोड़ा लाभ होना

थोड़ा-थोड़ा लाभ होना

C

बहुत बुरा लगना

बहुत बुरा लगना

D

थोड़ा-थोड़ा बचा कर रखना

थोड़ा-थोड़ा बचा कर रखना


Solution:

Correct Answer:

D

थोड़ा-थोड़ा बचा कर रखना

थोड़ा-थोड़ा बचा कर रखना


Question 131


निम्नलिखित में से किस विकल्प में लोकोक्ति का प्रयोग अशुद्ध है?

निम्नलिखित में से किस विकल्प में लोकोक्ति का प्रयोग अशुद्ध है?

Options

A

लड़के वालों के पास इतनी अकूत संपत्ति है फिर भी अपने लालच में लड़की के घर वालों से बहुत कुछ मिलने की आशा कर रहे हैं| इन पर तो यही कहावत चरितार्थ होती है कि ‘झोली डारे गज फिरे मुक्ता डारे साथ|’

लड़के वालों के पास इतनी अकूत संपत्ति है फिर भी अपने लालच में लड़की के घर वालों से बहुत कुछ मिलने की आशा कर रहे हैं| इन पर तो यही कहावत चरितार्थ होती है कि ‘झोली डारे गज फिरे मुक्ता डारे साथ|’

B

यहाँ के चपरासी के व्यवहार से सभी त्रस्त हैं लेकिन उसकी निर्लज्जता के कारण कोई उससे नहीं उलझता, किसी ने सच ही कहा है कि नंगा बड़ा परमेश्वर से|

यहाँ के चपरासी के व्यवहार से सभी त्रस्त हैं लेकिन उसकी निर्लज्जता के कारण कोई उससे नहीं उलझता, किसी ने सच ही कहा है कि नंगा बड़ा परमेश्वर से|

C

अभी मतदान को कई दिन बाकी हैं, ये जो कर रहे हैं करने दो, तुम तो बस ‘तेल देखो और तेल की धार देखो |’

अभी मतदान को कई दिन बाकी हैं, ये जो कर रहे हैं करने दो, तुम तो बस ‘तेल देखो और तेल की धार देखो |’

D

ज़रा सा काम था और पूरा दिन लगा दिया तुमने! इसे कहते हैं, ‘दिल्ली दूर है|’

ज़रा सा काम था और पूरा दिन लगा दिया तुमने! इसे कहते हैं, ‘दिल्ली दूर है|’


Solution:

Correct Answer:

D

ज़रा सा काम था और पूरा दिन लगा दिया तुमने! इसे कहते हैं, ‘दिल्ली दूर है|’

ज़रा सा काम था और पूरा दिन लगा दिया तुमने! इसे कहते हैं, ‘दिल्ली दूर है|’


Question 132


‘शेखी सेठ की धोती भाड़े की’– इस कहावत के उचित अर्थ का चयन करें-

‘शेखी सेठ की धोती भाड़े की’– इस कहावत के उचित अर्थ का चयन करें-

Options

A

कुछ न होने पर भी बड़प्पन दिखाना

कुछ न होने पर भी बड़प्पन दिखाना

B

ऊपर से अच्छा, मन से बुरा

ऊपर से अच्छा, मन से बुरा

C

शर्म करने से कष्ट उठाना पड़ता है

शर्म करने से कष्ट उठाना पड़ता है

D

सामर्थ्यवान के लिए कोई उपाय नहीं

सामर्थ्यवान के लिए कोई उपाय नहीं


Solution:

Correct Answer:

A

कुछ न होने पर भी बड़प्पन दिखाना

कुछ न होने पर भी बड़प्पन दिखाना


Question 133


‘गल्ला’ शब्द के पर्यायवाची का चयन करें-

‘गल्ला’ शब्द के पर्यायवाची का चयन करें-

Options

A

अनाज

अनाज

B

निवाला

निवाला

C

कन्दरा

कन्दरा

D

पुरवा

पुरवा


Solution:

Correct Answer:

A

अनाज

अनाज


Question 134


निम्नलिखित में से किस वाक्य में शब्द-चयन सम्बन्धी अशुद्धि नहीं है?

निम्नलिखित में से किस वाक्य में शब्द-चयन सम्बन्धी अशुद्धि नहीं है?

Options

A

प्रेम करना तलवार की नोंक पर चलना है|

प्रेम करना तलवार की नोंक पर चलना है|

B

उसने पुरस्कार का तिरस्कार कर दिया |

उसने पुरस्कार का तिरस्कार कर दिया |

C

यह एक अनुवादित निबंध है|

यह एक अनुवादित निबंध है|

D

कई दिनों की बदली के पीछे आज धूप निकली है|

कई दिनों की बदली के पीछे आज धूप निकली है|


Solution:

Correct Answer:

B

उसने पुरस्कार का तिरस्कार कर दिया |

उसने पुरस्कार का तिरस्कार कर दिया |


Question 135


निम्नलिखित वाक्य के अशुद्ध भाग को चिह्नित कीजिए –
‘यह भी तो संभव हो सकता है कि वह विद्यालय न जाकर अपने मित्रों के साथ खेलने चला गया हो|’

निम्नलिखित वाक्य के अशुद्ध भाग को चिह्नित कीजिए –
‘यह भी तो संभव हो सकता है कि वह विद्यालय न जाकर अपने मित्रों के साथ खेलने चला गया हो|’

Options

A

अपने मित्रों के साथ

अपने मित्रों के साथ

B

कि वह विद्यालय न जाकर

कि वह विद्यालय न जाकर

C

यह भी तो संभव हो सकता है

यह भी तो संभव हो सकता है

D

खेलने चला गया हो

खेलने चला गया हो


Solution:

Correct Answer:

C

यह भी तो संभव हो सकता है

यह भी तो संभव हो सकता है


Question 136


‘अरघट्ट’ शब्द के शुद्ध वर्तनी वाले तद्भव रूप का चयन कीजिए -

‘अरघट्ट’ शब्द के शुद्ध वर्तनी वाले तद्भव रूप का चयन कीजिए -

Options

A

रहट

रहट

B

सिंघाड़ा

सिंघाड़ा

C

घड़ा

घड़ा

D

रीठा

रीठा


Solution:

Correct Answer:

A

रहट

रहट


Question 137


निम्नलिखित विकल्पों में से ‘पार्वती’ के पर्यायवाची का चयन कीजिए-

निम्नलिखित विकल्पों में से ‘पार्वती’ के पर्यायवाची का चयन कीजिए-

Options

A

अपर्णा

अपर्णा

B

भारती

भारती

C

इंदिरा

इंदिरा

D

रमा

रमा


Solution:

Correct Answer:

A

अपर्णा

अपर्णा


Question 138


निम्नलिखित में से किस मुहावरे का अर्थ ‘झंझट सर ले लेना’ है?

निम्नलिखित में से किस मुहावरे का अर्थ ‘झंझट सर ले लेना’ है?

Options

A

कलेजे पर साँप लोटना

कलेजे पर साँप लोटना

B

इल्लत पालना

इल्लत पालना

C

इल्लत काटना

इल्लत काटना

D

इनायत होना

इनायत होना


Solution:

Correct Answer:

B

इल्लत पालना

इल्लत पालना


Question 139


निम्नलिखित वाक्यों में से अशुद्ध वाक्य का चयन कीजिये-

निम्नलिखित वाक्यों में से अशुद्ध वाक्य का चयन कीजिये-

Options

A

उसने घर आते ही सबको प्रणाम किया|

उसने घर आते ही सबको प्रणाम किया|

B

तुम लोग कब वापस लौटे?

तुम लोग कब वापस लौटे?

C

समझदार व्यक्ति के लिए संकेत मात्र ही पर्याप्त होता है|

समझदार व्यक्ति के लिए संकेत मात्र ही पर्याप्त होता है|

D

गिलास में दूध ऊपर तक भरा हुआ है|

गिलास में दूध ऊपर तक भरा हुआ है|


Solution:

Correct Answer:

B

तुम लोग कब वापस लौटे?

तुम लोग कब वापस लौटे?


Question 140


‘दूर की कौड़ी लाना’ मुहावरे के उचित अर्थ का चयन कीजिये-

‘दूर की कौड़ी लाना’ मुहावरे के उचित अर्थ का चयन कीजिये-

Options

A

कौड़ियों से खेलना

कौड़ियों से खेलना

B

ज्ञान और अनुभव न होना

ज्ञान और अनुभव न होना

C

दूर की सोच लेना

दूर की सोच लेना

D

दूर से बात करना

दूर से बात करना


Solution:

Correct Answer:

C

दूर की सोच लेना

दूर की सोच लेना


Question 141


निम्नलिखित में से किस कथन के लिए ‘रातों रोई एक ही मुआ’ कहावत उपयुक्त है?

निम्नलिखित में से किस कथन के लिए ‘रातों रोई एक ही मुआ’ कहावत उपयुक्त है?

Options

A

रोने से क्या होगा, जो होना था सो हुआ, भाग्य की बात कोई नहीं टाल सकत, ____

रोने से क्या होगा, जो होना था सो हुआ, भाग्य की बात कोई नहीं टाल सकत, ____

B

इतना समझाया लेकिन उसे अपनी बुराई या भूल समझ में ही नहीं आ रही है, ____

इतना समझाया लेकिन उसे अपनी बुराई या भूल समझ में ही नहीं आ रही है, ____

C

दुष्ट लोग समझाने से वश में नहीं आते, उनके लिए कठोर दण्ड ही उचित है, ____

दुष्ट लोग समझाने से वश में नहीं आते, उनके लिए कठोर दण्ड ही उचित है, ____

D

मैंने इतनी मेहनत से एक सप्ताह तक प्रतिदिन रमा को समास पढ़ाया, लेकिन ________, उसकी समझ में सिर्फ इतना ही आया कि दो शब्द मिलकर समास बनते हैं |

मैंने इतनी मेहनत से एक सप्ताह तक प्रतिदिन रमा को समास पढ़ाया, लेकिन ________, उसकी समझ में सिर्फ इतना ही आया कि दो शब्द मिलकर समास बनते हैं |


Solution:

Correct Answer:

D

मैंने इतनी मेहनत से एक सप्ताह तक प्रतिदिन रमा को समास पढ़ाया, लेकिन ________, उसकी समझ में सिर्फ इतना ही आया कि दो शब्द मिलकर समास बनते हैं |

मैंने इतनी मेहनत से एक सप्ताह तक प्रतिदिन रमा को समास पढ़ाया, लेकिन ________, उसकी समझ में सिर्फ इतना ही आया कि दो शब्द मिलकर समास बनते हैं |


Question 142


निम्नलिखित में से किस विकल्प के दोनों शब्द आपस में पर्यायवाची हैं?

निम्नलिखित में से किस विकल्प के दोनों शब्द आपस में पर्यायवाची हैं?

Options

A

वह्नि, हुताशन

वह्नि, हुताशन

B

कंज, छाप

कंज, छाप

C

अकिंचन, स्थावर

अकिंचन, स्थावर

D

विलक्षण, अजंगम

विलक्षण, अजंगम


Solution:

Correct Answer:

A

वह्नि, हुताशन

वह्नि, हुताशन


Question 143


‘भैस के आगे बीन बजाना’ मुहावरे के समानार्थी मुहावरे का चयन करें-

‘भैस के आगे बीन बजाना’ मुहावरे के समानार्थी मुहावरे का चयन करें-

Options

A

मौत को दुलत्ती मारना

मौत को दुलत्ती मारना

B

मोह की धार में बहना

मोह की धार में बहना

C

मुर्दे को जिंदा करना

मुर्दे को जिंदा करना

D

मुर्गी के आगे हीरा रखना

मुर्गी के आगे हीरा रखना


Solution:

Correct Answer:

D

मुर्गी के आगे हीरा रखना

मुर्गी के आगे हीरा रखना


Question 144


सटीक मुहावरा चुनकर, निम्नलिखित वाक्य की पूर्ति कीजिये-
वाक्य- ‘आप बिलकुल सही कह रहे हैं, वैसे भी आपकी सभी बातें ______ हुआ करती हैं|’

सटीक मुहावरा चुनकर, निम्नलिखित वाक्य की पूर्ति कीजिये-
वाक्य- ‘आप बिलकुल सही कह रहे हैं, वैसे भी आपकी सभी बातें ______ हुआ करती हैं|’

Options

A

बावले की जड़

बावले की जड़

B

बालू की भीत

बालू की भीत

C

बावन तोले पाँव रत्ती

बावन तोले पाँव रत्ती

D

बावन–बीर

बावन–बीर


Solution:

Correct Answer:

C

बावन तोले पाँव रत्ती

बावन तोले पाँव रत्ती


Question 145


निम्नलिखित वाक्यों में से अशुद्ध वाक्य की पहचान कीजिये-

निम्नलिखित वाक्यों में से अशुद्ध वाक्य की पहचान कीजिये-

Options

A

कुछ अतिथि आ चुके हैं, उन्हें जलपान कराना है|

कुछ अतिथि आ चुके हैं, उन्हें जलपान कराना है|

B

यदि कोई मित्र उनके पास आता है तो वे उसे बैठने भी नहीं देते|

यदि कोई मित्र उनके पास आता है तो वे उसे बैठने भी नहीं देते|

C

मनुष्य ने प्रकृति पर विजय पा ली है|

मनुष्य ने प्रकृति पर विजय पा ली है|

D

इस तरह की बातें सुनते-सुनते मेरा कान पक गया|

इस तरह की बातें सुनते-सुनते मेरा कान पक गया|


Solution:

Correct Answer:

D

इस तरह की बातें सुनते-सुनते मेरा कान पक गया|

इस तरह की बातें सुनते-सुनते मेरा कान पक गया|


Question 146


‘तुमने मुझसे आग्रह किया कि इसे गुप्त रहस्य ही रहने दीजिए ’
उपर्युक्त वाक्य के किस भाग में अशुद्धि है?

‘तुमने मुझसे आग्रह किया कि इसे गुप्त रहस्य ही रहने दीजिए ’
उपर्युक्त वाक्य के किस भाग में अशुद्धि है?

Options

A

गुप्त रहस्य ही

गुप्त रहस्य ही

B

रहने दीजिए

रहने दीजिए

C

किया कि इसे

किया कि इसे

D

तुमने मुझसे आग्रह

तुमने मुझसे आग्रह


Solution:

Correct Answer:

A

गुप्त रहस्य ही

गुप्त रहस्य ही


Question 147


‘एकांगी’ के विलोम युग्म की पहचान करें-

‘एकांगी’ के विलोम युग्म की पहचान करें-

Options

A

अनेकत्व

अनेकत्व

B

दुचित्ता

दुचित्ता

C

सर्वांगीण

सर्वांगीण

D

तिर्यक

तिर्यक


Solution:

Correct Answer:

C

सर्वांगीण

सर्वांगीण


Question 148


निम्नलिखित में से किस वाक्य में ‘कर्म और क्रिया’ का अन्वय अशुद्ध है?

निम्नलिखित में से किस वाक्य में ‘कर्म और क्रिया’ का अन्वय अशुद्ध है?

Options

A

अनुपमा के सभी भाई या पाँच बहनें शादी में जाएँगी|

अनुपमा के सभी भाई या पाँच बहनें शादी में जाएँगी|

B

इस सम्मलेन में बहुत से लड़के और लड़कियाँ आए|

इस सम्मलेन में बहुत से लड़के और लड़कियाँ आए|

C

मैंने बाज़ार से कुरता और चप्पल खरीदा|

मैंने बाज़ार से कुरता और चप्पल खरीदा|

D

निशा ने आज कमला से क्षमा माँगी|

निशा ने आज कमला से क्षमा माँगी|


Solution:

Correct Answer:

C

मैंने बाज़ार से कुरता और चप्पल खरीदा|

मैंने बाज़ार से कुरता और चप्पल खरीदा|


Question 149


‘तुम्हें कितना समझाया-बुझाया कि शहर जाकर कुछ काम कर लो, लेकिन परिणाम ______, तुम कुछ नहीं करोगे|
उपयुक्त लोकोक्ति चुनकर, वाक्य में रिक्त स्थान की पूर्ति कीजिये-

‘तुम्हें कितना समझाया-बुझाया कि शहर जाकर कुछ काम कर लो, लेकिन परिणाम ______, तुम कुछ नहीं करोगे|
उपयुक्त लोकोक्ति चुनकर, वाक्य में रिक्त स्थान की पूर्ति कीजिये-

Options

A

तिनके की ओट में पहाड़

तिनके की ओट में पहाड़

B

तू डाल-डाल तो मैं पात-पात

तू डाल-डाल तो मैं पात-पात

C

छोटा मुँह बड़ी बात

छोटा मुँह बड़ी बात

D

वही ढाक के तीन पात

वही ढाक के तीन पात


Solution:

Correct Answer:

D

वही ढाक के तीन पात

वही ढाक के तीन पात


Question 150


‘हंसा थे सो उड़ गए, कागा भये दीवान’ – इस लोकोक्ति के उचित अर्थ का चयन करें-

‘हंसा थे सो उड़ गए, कागा भये दीवान’ – इस लोकोक्ति के उचित अर्थ का चयन करें-

Options

A

कठिनाइयाँ झेल कर ही आदमी ऊँचा पद पाता है

कठिनाइयाँ झेल कर ही आदमी ऊँचा पद पाता है

B

बड़ों के रहते छोटों को क्या पूछना

बड़ों के रहते छोटों को क्या पूछना

C

गुणी व्यक्ति का गुण हर हाल में प्रकट हो जाता है

गुणी व्यक्ति का गुण हर हाल में प्रकट हो जाता है

D

भले लोगों के स्थान पर बुरे लोगों के हाथ में अधिकार

भले लोगों के स्थान पर बुरे लोगों के हाथ में अधिकार


Solution:

Correct Answer:

D

भले लोगों के स्थान पर बुरे लोगों के हाथ में अधिकार

भले लोगों के स्थान पर बुरे लोगों के हाथ में अधिकार


Question 151


तत्सम शब्द ‘सप्तशती’ के तद्भव रूप का चयन कीजिए -

तत्सम शब्द ‘सप्तशती’ के तद्भव रूप का चयन कीजिए -

Options

A

सतहत्तर

सतहत्तर

B

सप्तदश

सप्तदश

C

सतसई

सतसई

D

सत्तासी

सत्तासी


Solution:

Correct Answer:

C

सतसई

सतसई


Question 152


निम्नलिखित में से किस वाक्य में ‘कर्ता और क्रिया’ का अन्वय शुद्ध है?

निम्नलिखित में से किस वाक्य में ‘कर्ता और क्रिया’ का अन्वय शुद्ध है?

Options

A

चंचल और मुन्ना मुम्बई चली गयी हैं|

चंचल और मुन्ना मुम्बई चली गयी हैं|

B

कुछ लड़के, दो वृद्धाएँ और कुछ बालिकाएँ मैदान में बातें कर रहे हैं|

कुछ लड़के, दो वृद्धाएँ और कुछ बालिकाएँ मैदान में बातें कर रहे हैं|

C

बिल्ली ने दूध और मलाई खाया |

बिल्ली ने दूध और मलाई खाया |

D

रमा, शीला, सुरेश और तितली आये और वे चले भी गए|

रमा, शीला, सुरेश और तितली आये और वे चले भी गए|


Solution:

Correct Answer:

D

रमा, शीला, सुरेश और तितली आये और वे चले भी गए|

रमा, शीला, सुरेश और तितली आये और वे चले भी गए|


Question 153


‘पीला’ शब्द के उचित पर्यायवाची का चयन करें-

‘पीला’ शब्द के उचित पर्यायवाची का चयन करें-

Options

A

पिंगल

पिंगल

B

पीयूष

पीयूष

C

पीवर

पीवर

D

पुंगव

पुंगव


Solution:

Correct Answer:

A

पिंगल

पिंगल


Question 154


‘एक तो आदित्य निठल्ले किस्म का व्यक्ति है, उस पर घर वालों के साथ इतना दुर्व्यवहार बिलकुल ऐसा ही है जैसे ‘इतना खाए, जितना पचे|’
उपर्युक्त वाक्य में कौन सा भाग असंगत है? चिह्नित कीजिये-

‘एक तो आदित्य निठल्ले किस्म का व्यक्ति है, उस पर घर वालों के साथ इतना दुर्व्यवहार बिलकुल ऐसा ही है जैसे ‘इतना खाए, जितना पचे|’
उपर्युक्त वाक्य में कौन सा भाग असंगत है? चिह्नित कीजिये-

Options

A

एक तो आदित्य निठल्ले किस्म का व्यक्ति है,

एक तो आदित्य निठल्ले किस्म का व्यक्ति है,

B

इतना दुर्व्यवहार बिलकुल ऐसा ही है

इतना दुर्व्यवहार बिलकुल ऐसा ही है

C

जैसे ‘इतना खाए, जितना पचे|’

जैसे ‘इतना खाए, जितना पचे|’

D

उस पर घर वालों के साथ

उस पर घर वालों के साथ


Solution:

Correct Answer:

C

जैसे ‘इतना खाए, जितना पचे|’

जैसे ‘इतना खाए, जितना पचे|’


Question 155


निम्नलिखित में से किस विकल्प में मुहावरे का प्रयोग अशुद्ध है?

निम्नलिखित में से किस विकल्प में मुहावरे का प्रयोग अशुद्ध है?

Options

A

तुम्हारा हर काम कुल्हिया में गुड़ फोड़ने जैसा है, कुछ पता ही नहीं चलता कि कब हो गया|

तुम्हारा हर काम कुल्हिया में गुड़ फोड़ने जैसा है, कुछ पता ही नहीं चलता कि कब हो गया|

B

इस वैज्ञानिक युग में अनोखे आविष्कार करके संसार बादल में थिगली लगा रहा है|

इस वैज्ञानिक युग में अनोखे आविष्कार करके संसार बादल में थिगली लगा रहा है|

C

पुलिस को देखते ही चोरों ने यहाँ से केंचुली बदल ली|

पुलिस को देखते ही चोरों ने यहाँ से केंचुली बदल ली|

D

आज तुम्हारी सुन्दरता देख कर कहना पड़ रहा है कि तुमने बाल-बाल गजमोती पिरोया है |

आज तुम्हारी सुन्दरता देख कर कहना पड़ रहा है कि तुमने बाल-बाल गजमोती पिरोया है |


Solution:

Correct Answer:

C

पुलिस को देखते ही चोरों ने यहाँ से केंचुली बदल ली|

पुलिस को देखते ही चोरों ने यहाँ से केंचुली बदल ली|


Question 156


‘ऊसर’ शब्द के पर्यायवाची का चयन कीजिए -

‘ऊसर’ शब्द के पर्यायवाची का चयन कीजिए -

Options

A

सस्यहीन

सस्यहीन

B

उर्वर

उर्वर

C

ऊष्ण

ऊष्ण

D

सूना

सूना


Solution:

Correct Answer:

A

सस्यहीन

सस्यहीन


Question 157


निम्नलिखित में से ‘प्राची’ के विलोमार्थी शब्द का चयन करें-

निम्नलिखित में से ‘प्राची’ के विलोमार्थी शब्द का चयन करें-

Options

A

दक्षिण

दक्षिण

B

कृत्रिम

कृत्रिम

C

पश्चिम

पश्चिम

D

अंतिम

अंतिम


Solution:

Correct Answer:

C

पश्चिम

पश्चिम


Question 158


निम्नलिखित में से कौन-सा शब्द ‘ईषत्’ का विपरीतार्थक है?

निम्नलिखित में से कौन-सा शब्द ‘ईषत्’ का विपरीतार्थक है?

Options

A

ईहा

ईहा

B

प्रचुर

प्रचुर

C

जीव

जीव

D

प्रयत्न

प्रयत्न


Solution:

Correct Answer:

B

प्रचुर

प्रचुर


Question 159


निम्नलिखित में से कौन सा शब्द ‘त्र्यंबक’ का पर्यायवाची नहीं है?

निम्नलिखित में से कौन सा शब्द ‘त्र्यंबक’ का पर्यायवाची नहीं है?

Options

A

त्रिचक्षु

त्रिचक्षु

B

विरूपाक्ष

विरूपाक्ष

C

त्रिदृश

त्रिदृश

D

त्रिलोक

त्रिलोक


Solution:

Correct Answer:

D

त्रिलोक

त्रिलोक


Question 160


निम्नलिखित में से किस वाक्य में लिंग सम्बन्धी अशुद्धि है?

निम्नलिखित में से किस वाक्य में लिंग सम्बन्धी अशुद्धि है?

Options

A

काशी सदैव से भारतीय संस्कृति का केंद्र रहा है|

काशी सदैव से भारतीय संस्कृति का केंद्र रहा है|

B

प्राचीनकाल में साहित्यकार ज्वलंत सामाजिक समस्याओं पर अपना साहित्य प्रस्तुत करते थे|

प्राचीनकाल में साहित्यकार ज्वलंत सामाजिक समस्याओं पर अपना साहित्य प्रस्तुत करते थे|

C

तुम्हारी इन्हीं आदतों के कारण मुझे बातें सुननी पड़ती हैं|

तुम्हारी इन्हीं आदतों के कारण मुझे बातें सुननी पड़ती हैं|

D

अच्छा उपन्यास पाठक में पढ़ने की रुचि पैदा करता है|

अच्छा उपन्यास पाठक में पढ़ने की रुचि पैदा करता है|


Solution:

Correct Answer:

A

काशी सदैव से भारतीय संस्कृति का केंद्र रहा है|

काशी सदैव से भारतीय संस्कृति का केंद्र रहा है|


Question 161


‘कविता पढ़ने और नाटक देखने से पाठक और श्रोता को जो असाधारण और अनिर्वचनीय आनंद की अनुभूति होती है, वही रस है|’
उपर्युक्त वाक्य के किस अंश में त्रुटि है? पहचान करें-

‘कविता पढ़ने और नाटक देखने से पाठक और श्रोता को जो असाधारण और अनिर्वचनीय आनंद की अनुभूति होती है, वही रस है|’
उपर्युक्त वाक्य के किस अंश में त्रुटि है? पहचान करें-

Options

A

कविता पढ़ने और नाटक देखने से

कविता पढ़ने और नाटक देखने से

B

असाधारण और अनिर्वचनीय आनंद की

असाधारण और अनिर्वचनीय आनंद की

C

अनुभूति होती है, वही रस है

अनुभूति होती है, वही रस है

D

पाठक और श्रोता को जो

पाठक और श्रोता को जो


Solution:

Correct Answer:

D

पाठक और श्रोता को जो

पाठक और श्रोता को जो


Question 162


‘बहुत समय पहले कलकत्ता ही भारत की राजधानी थी|’
उपर्युक्त वाक्य के अशुद्ध भाग का चयन कीजिये-

‘बहुत समय पहले कलकत्ता ही भारत की राजधानी थी|’
उपर्युक्त वाक्य के अशुद्ध भाग का चयन कीजिये-

Options

A

बहुत समय पहले

बहुत समय पहले

B

भारत की

भारत की

C

राजधानी थी

राजधानी थी

D

कलकत्ता ही

कलकत्ता ही


Solution:

Correct Answer:

C

राजधानी थी

राजधानी थी


Question 163


सटीक कहावत चुनकर, निम्नलिखित कथन की पूर्ति कीजिये-
वाक्य- ‘आज रेशमा दूध फ़्रिज में रखना भूल गयी और दूध फट गया| जब उसकी माँ ने डाँटा तो जल्दी से बोली कि मैंने पनीर बनाने के लिए जानबूझ कर फाड़ा है, वाह! वाह! _______ |

सटीक कहावत चुनकर, निम्नलिखित कथन की पूर्ति कीजिये-
वाक्य- ‘आज रेशमा दूध फ़्रिज में रखना भूल गयी और दूध फट गया| जब उसकी माँ ने डाँटा तो जल्दी से बोली कि मैंने पनीर बनाने के लिए जानबूझ कर फाड़ा है, वाह! वाह! _______ |

Options

A

फिसल पड़े तो हर गंगा

फिसल पड़े तो हर गंगा

B

बहती गंगा में हाथ धो लिया

बहती गंगा में हाथ धो लिया

C

फलूदा खाते दाँत टूटें तो टूटें

फलूदा खाते दाँत टूटें तो टूटें

D

पकाई खीर पर हो गया दलिया

पकाई खीर पर हो गया दलिया


Solution:

Correct Answer:

A

फिसल पड़े तो हर गंगा

फिसल पड़े तो हर गंगा


Question 164


निम्नलिखित में से अशुद्ध वर्तनी वाले तत्सम शब्द का चयन करें-

निम्नलिखित में से अशुद्ध वर्तनी वाले तत्सम शब्द का चयन करें-

Options

A

लश्शुन

लश्शुन

B

बैराग

बैराग

C

जनेऊ

जनेऊ

D

भौंरा

भौंरा


Solution:

Correct Answer:

A

लश्शुन

लश्शुन


Question 165


निम्नलिखित में से तत्सम शब्द की पहचान कीजिए -

निम्नलिखित में से तत्सम शब्द की पहचान कीजिए -

Options

A

लौंग

लौंग

B

भ्रम

भ्रम

C

लज्जा

लज्जा

D

वन

वन


Solution:

Correct Answer:

B

भ्रम

भ्रम


Question 166


निम्नलिखित वाक्यों में से शुद्ध वाक्य का चयन कीजिये-

निम्नलिखित वाक्यों में से शुद्ध वाक्य का चयन कीजिये-

Options

A

बिल गेट्स ने अपनी कमाई का अधिकांश भाग दान में दे दिया|

बिल गेट्स ने अपनी कमाई का अधिकांश भाग दान में दे दिया|

B

बिल गेट्स ने अपने कमाई का अधिकांश भाग दान में दे दिया|

बिल गेट्स ने अपने कमाई का अधिकांश भाग दान में दे दिया|

C

बिल गेट्स ने अपनी कमाई का अधिकांश दान कर दिया|

बिल गेट्स ने अपनी कमाई का अधिकांश दान कर दिया|

D

बिल गेट्स ने अपने कमाई का अधिकांश भाग दान कर दिया|

बिल गेट्स ने अपने कमाई का अधिकांश भाग दान कर दिया|


Solution:

Correct Answer:

C

बिल गेट्स ने अपनी कमाई का अधिकांश दान कर दिया|

बिल गेट्स ने अपनी कमाई का अधिकांश दान कर दिया|


Question 167


निम्नलिखित वाक्य के अशुद्ध भाग का चयन कीजिये-
‘इस गली में ताजा गाय का दूध और शुद्ध घी मिलता है|’

निम्नलिखित वाक्य के अशुद्ध भाग का चयन कीजिये-
‘इस गली में ताजा गाय का दूध और शुद्ध घी मिलता है|’

Options

A

इस गली में

इस गली में

B

मिलता है

मिलता है

C

और शुद्ध घी

और शुद्ध घी

D

ताजा गाय का दूध

ताजा गाय का दूध


Solution:

Correct Answer:

D

ताजा गाय का दूध

ताजा गाय का दूध


Question 168


निम्नलिखित में से ‘म्याऊँ का ठौर पकड़ना’ मुहावरे का उचित अर्थ क्या है?

निम्नलिखित में से ‘म्याऊँ का ठौर पकड़ना’ मुहावरे का उचित अर्थ क्या है?

Options

A

लड़ने को तैयार होना

लड़ने को तैयार होना

B

लड़ाई जीत जाना

लड़ाई जीत जाना

C

पुष्टि करना

पुष्टि करना

D

खतरे में पड़ना

खतरे में पड़ना


Solution:

Correct Answer:

D

खतरे में पड़ना

खतरे में पड़ना


Question 169


निम्नलिखित में से तत्सम शब्द की पहचान कीजिए -

निम्नलिखित में से तत्सम शब्द की पहचान कीजिए -

Options

A

भट्ठी

भट्ठी

B

अभ्यंतर

अभ्यंतर

C

मच्छर

मच्छर

D

बत्ती

बत्ती


Solution:

Correct Answer:

B

अभ्यंतर

अभ्यंतर


Question 170


निम्नलिखित में से किस विकल्प के सभी शब्द ‘लहर’ के पर्याय हैं?

निम्नलिखित में से किस विकल्प के सभी शब्द ‘लहर’ के पर्याय हैं?

Options

A

उमंग, उर्मिल

उमंग, उर्मिल

B

मौज, विभावरी

मौज, विभावरी

C

हिलोर, यामा

हिलोर, यामा

D

उर्मि, वीचि

उर्मि, वीचि


Solution:

Correct Answer:

D

उर्मि, वीचि

उर्मि, वीचि


Question 171


‘विद्यालय सभागार में आयोजित इस समारोह में शहर के अनेकों गणमान्य लोग उपस्थित होंगे|’
उपर्युक्त वाक्य के किस भाग में अशुद्धि है?

‘विद्यालय सभागार में आयोजित इस समारोह में शहर के अनेकों गणमान्य लोग उपस्थित होंगे|’
उपर्युक्त वाक्य के किस भाग में अशुद्धि है?

Options

A

विद्यालय सभागार में आयोजित

विद्यालय सभागार में आयोजित

B

इस समारोह में

इस समारोह में

C

शहर के अनेकों गणमान्य लोग

शहर के अनेकों गणमान्य लोग

D

उपस्थित होंगे

उपस्थित होंगे


Solution:

Correct Answer:

C

शहर के अनेकों गणमान्य लोग

शहर के अनेकों गणमान्य लोग


Question 172


निम्नलिखित में से कौन सा विलोम- युग्म शुद्ध है?

निम्नलिखित में से कौन सा विलोम- युग्म शुद्ध है?

Options

A

उल्लास – उद्वेग

उल्लास – उद्वेग

B

ऋद्ध – वृद्ध

ऋद्ध – वृद्ध

C

कुंचन – लुंचन

कुंचन – लुंचन

D

ऋत – अनृत

ऋत – अनृत


Solution:

Correct Answer:

D

ऋत – अनृत

ऋत – अनृत


Question 173


निम्नलिखित शब्द के शुद्ध तत्सम रूप का चयन करें-
उलाहना

निम्नलिखित शब्द के शुद्ध तत्सम रूप का चयन करें-
उलाहना

Options

A

उलालंभ

उलालंभ

B

उल्लहन

उल्लहन

C

उलक्ष्णा

उलक्ष्णा

D

उपालंभ

उपालंभ


Solution:

Correct Answer:

D

उपालंभ

उपालंभ


Question 174


निम्नलिखित में से अशुद्ध विलोम- युग्म की पहचान करें-

निम्नलिखित में से अशुद्ध विलोम- युग्म की पहचान करें-

Options

A

कनिष्ठ – ज्येष्ठ

कनिष्ठ – ज्येष्ठ

B

उग्र – सौम्य

उग्र – सौम्य

C

अक्ष – अर्क

अक्ष – अर्क

D

क्षणिक – शाश्वत

क्षणिक – शाश्वत


Solution:

Correct Answer:

C

अक्ष – अर्क

अक्ष – अर्क


Question 175


निम्नलिखित में से ‘तृष्णा’ शब्द के उचित विलोमार्थी का चयन करें-

निम्नलिखित में से ‘तृष्णा’ शब्द के उचित विलोमार्थी का चयन करें-

Options

A

तृषा

तृषा

B

वितृष्णा

वितृष्णा

C

अपतृष्णा

अपतृष्णा

D

अतृष्णा

अतृष्णा


Solution:

Correct Answer:

B

वितृष्णा

वितृष्णा


Question 176


Four sentences of a paragraph are given below in jumbled order. Arrange the sentences in the correct order to form a meaningful and coherent paragraph.

(A) Whenever I teach my class, I make sure that everyone participates in the question-answer session.
(B) This way, students learn to express their opinions in front of others and the teacher also understands how much the students have learnt.
(C) I normally teach for forty minutes and the last twenty minutes I keep open for debates, questions and answers.
(D) Many times I have learnt a lot from my students during these sessions.

Four sentences of a paragraph are given below in jumbled order. Arrange the sentences in the correct order to form a meaningful and coherent paragraph.

(A) Whenever I teach my class, I make sure that everyone participates in the question-answer session.
(B) This way, students learn to express their opinions in front of others and the teacher also understands how much the students have learnt.
(C) I normally teach for forty minutes and the last twenty minutes I keep open for debates, questions and answers.
(D) Many times I have learnt a lot from my students during these sessions.

Options

A

ACBD

ACBD

B

DACB

DACB

C

ADBC

ADBC

D

CBDA

CBDA


Solution:

Correct Answer:

A

ACBD

ACBD


Question 177


Select the most appropriate option to fill in the blank.

How long can you __________ in this cold weather without proper clothing?

Select the most appropriate option to fill in the blank.

How long can you __________ in this cold weather without proper clothing?

Options

A

hold with

hold with

B

hold up

hold up

C

hold back

hold back

D

hold on

hold on


Solution:

Correct Answer:

D

hold on

hold on


Question 178


Select the most appropriate option to fill in the blank.

He sketched the danger of death and destruction that __________ hung over the world.

Select the most appropriate option to fill in the blank.

He sketched the danger of death and destruction that __________ hung over the world.

Options

A

still

still

B

rather

rather

C

quite

quite

D

enough

enough


Solution:

Correct Answer:

A

still

still


Question 179


Segments of the following sentence have been given as options. One of them may have a spelling error. Select the segment that contains the error. If you don’t find any error, mark ‘No error’ as your answer.

Modern industrialised communities experience the joy which nature gives on ocassional weekends.

Segments of the following sentence have been given as options. One of them may have a spelling error. Select the segment that contains the error. If you don’t find any error, mark ‘No error’ as your answer.

Modern industrialised communities experience the joy which nature gives on ocassional weekends.

Options

A

Modern industrialised communities

Modern industrialised communities

B

No error

No error

C

experience the joy which nature gives

experience the joy which nature gives

D

on ocassional weekends.

on ocassional weekends.


Solution:

Correct Answer:

D

on ocassional weekends.

on ocassional weekends.


Question 180


Select the most appropriate synonym of the given word.

Incessant

Select the most appropriate synonym of the given word.

Incessant

Options

A

Halting

Halting

B

Ceaseless

Ceaseless

C

Infrequent

Infrequent

D

Broken

Broken


Solution:

Correct Answer:

B

Ceaseless

Ceaseless


Question 181


Select the most appropriate option to fill in the blank.

The poor girl did nothing __________ cry.

Select the most appropriate option to fill in the blank.

The poor girl did nothing __________ cry.

Options

A

so

so

B

as

as

C

but

but

D

and

and


Solution:

Correct Answer:

C

but

but


Question 182


Given below are four sentences, out of which three are connected. Select the odd sentence.

Given below are four sentences, out of which three are connected. Select the odd sentence.

Options

A

The identity of his killers and the location of his body are ongoing mysteries.

The identity of his killers and the location of his body are ongoing mysteries.

B

On 22 November 1963, President John F Kennedy was shot in Dallas by Lee Harvey Oswald. 

On 22 November 1963, President John F Kennedy was shot in Dallas by Lee Harvey Oswald. 

C

The most widely accepted explanation is that Oswald killed JFK on his own and Ruby killed Oswald, on his own volition.

The most widely accepted explanation is that Oswald killed JFK on his own and Ruby killed Oswald, on his own volition.

D

On 24 November 1963, before Oswald could stand trial, Oswald was fatally shot by nightclub owner Jack Ruby. 

On 24 November 1963, before Oswald could stand trial, Oswald was fatally shot by nightclub owner Jack Ruby. 


Solution:

Correct Answer:

A

The identity of his killers and the location of his body are ongoing mysteries.

The identity of his killers and the location of his body are ongoing mysteries.


Question 183


Select the option that expresses the given sentence in passive voice.

Havildar Abdul Hamid knocked out the oncoming enemy tank.

Select the option that expresses the given sentence in passive voice.

Havildar Abdul Hamid knocked out the oncoming enemy tank.

Options

A

The oncoming enemy tank was knocked out by Havildar Abdul Hamid.

The oncoming enemy tank was knocked out by Havildar Abdul Hamid.

B

The oncoming enemy tank will be knocked out by Havildar Abdul Hamid.

The oncoming enemy tank will be knocked out by Havildar Abdul Hamid.

C

Havildar Abdul Hamid was knocked out by the oncoming enemy tank.

Havildar Abdul Hamid was knocked out by the oncoming enemy tank.

D

The oncoming enemy tank had been knocked out by Havildar Abdul Hamid.

The oncoming enemy tank had been knocked out by Havildar Abdul Hamid.


Solution:

Correct Answer:

A

The oncoming enemy tank was knocked out by Havildar Abdul Hamid.

The oncoming enemy tank was knocked out by Havildar Abdul Hamid.


Question 184


Select the option that completes the given proverb correctly.

________ can’t be choosers.

Select the option that completes the given proverb correctly.

________ can’t be choosers.

Options

A

Paupers

Paupers

B

Bankrupts

Bankrupts

C

Vagrants

Vagrants

D

Beggars

Beggars


Solution:

Correct Answer:

D

Beggars

Beggars


Question 185


Sentences of a paragraph are given below in jumbled order. Arrange the sentences in the correct order to form a meaningful and coherent paragraph.

A. This socialising habit may have been an important factor for their brains to grow into a relatively large and complex organ.
B. Unlike many other species, dolphins live in complex social groups.
C. Dolphins are known to be highly intelligent, playful and friendly.
D.Scientists believe that it may have taken thousands of years for Dolphin brains to adapt to survival modes and to develop to their present level of intelligence.

Sentences of a paragraph are given below in jumbled order. Arrange the sentences in the correct order to form a meaningful and coherent paragraph.

A. This socialising habit may have been an important factor for their brains to grow into a relatively large and complex organ.
B. Unlike many other species, dolphins live in complex social groups.
C. Dolphins are known to be highly intelligent, playful and friendly.
D.Scientists believe that it may have taken thousands of years for Dolphin brains to adapt to survival modes and to develop to their present level of intelligence.

Options

A

DACB

DACB

B

CDAB

CDAB

C

ADBC

ADBC

D

CBAD

CBAD


Solution:

Correct Answer:

D

CBAD

CBAD


Question 186


Select the most appropriate synonym of the given word.

Extricate

Select the most appropriate synonym of the given word.

Extricate

Options

A

Detach

Detach

B

Remain

Remain

C

Entangle

Entangle

D

Connect

Connect


Solution:

Correct Answer:

A

Detach

Detach


Question 187


Select the correctly spelt word to fill in the blank.

She got an opportunity to participate in the painting exhibition by ______ artists.

Select the correctly spelt word to fill in the blank.

She got an opportunity to participate in the painting exhibition by ______ artists.

Options

A

amateur

amateur

B

amatuer

amatuer

C

ameteur

ameteur

D

amature

amature


Solution:

Correct Answer:

A

amateur

amateur


Question 188


Select the option that is NOT an ANTONYM of another word by way of adding the prefix ‘Dis-’.

Select the option that is NOT an ANTONYM of another word by way of adding the prefix ‘Dis-’.

Options

A

Discretion

Discretion

B

Discomfort

Discomfort

C

Disconnect

Disconnect

D

Discontinue

Discontinue


Solution:

Correct Answer:

A

Discretion

Discretion


Question 189


Select the most appropriate ANTONYM of the given word.

Queer

Select the most appropriate ANTONYM of the given word.

Queer

Options

A

Baffling

Baffling

B

Rare

Rare

C

Ordinary

Ordinary

D

Striking

Striking


Solution:

Correct Answer:

C

Ordinary

Ordinary


Question 190


Given below are four sentences, out of which three are connected. Select the odd sentence.

Given below are four sentences, out of which three are connected. Select the odd sentence.

Options

A

Malcolm drove Barb to a jewellery shop and they chose an engagement ring.

Malcolm drove Barb to a jewellery shop and they chose an engagement ring.

B

Surgery began with a seven hour emergency operation.

Surgery began with a seven hour emergency operation.

C

Today Malcolm and Barb live near Vancouver.

Today Malcolm and Barb live near Vancouver.

D

They were married on 27 July 2021.

They were married on 27 July 2021.


Solution:

Correct Answer:

B

Surgery began with a seven hour emergency operation.

Surgery began with a seven hour emergency operation.


Question 191


Select the most appropriate option that collocates with the word ‘dogs’ to fill in the blank.

The street dogs made quite a ruckus when they started _______ together last night.

Select the most appropriate option that collocates with the word ‘dogs’ to fill in the blank.

The street dogs made quite a ruckus when they started _______ together last night.

Options

A

neighing

neighing

B

roaring

roaring

C

barking

barking

D

braying

braying


Solution:

Correct Answer:

C

barking

barking


Question 192


Select the most appropriate option to fill in the blank.

_______ looking after the old lady, I also do their cooking.

Select the most appropriate option to fill in the blank.

_______ looking after the old lady, I also do their cooking.

Options

A

Although

Although

B

In spite of

In spite of

C

Besides

Besides

D

In case

In case


Solution:

Correct Answer:

C

Besides

Besides


Question 193


Select the most appropriate option that can substitute the underlined word in the given sentence. If there is no need to substitute it, select ‘No substitution required’.

She enjoys great punctuality among her students.

Select the most appropriate option that can substitute the underlined word in the given sentence. If there is no need to substitute it, select ‘No substitution required’.

She enjoys great punctuality among her students.

Options

A

popularity

popularity

B

perpetuity

perpetuity

C

No substitution required

No substitution required

D

partiality

partiality


Solution:

Correct Answer:

A

popularity

popularity


Question 194


Select the most appropriate option that can substitute the underlined word in the given sentence. If there is no need to substitute it, select ‘No substitution required’.

When are you going to launch solar panels on your roof top?

Select the most appropriate option that can substitute the underlined word in the given sentence. If there is no need to substitute it, select ‘No substitution required’.

When are you going to launch solar panels on your roof top?

Options

A

install

install

B

lift off

lift off

C

present

present

D

No substitution required

No substitution required


Solution:

Correct Answer:

A

install

install


Question 195


Select the most appropriate option to fill in the blanks.

This factory manufactures rubber __________. The employees have to __________ special suits.

Select the most appropriate option to fill in the blanks.

This factory manufactures rubber __________. The employees have to __________ special suits.

Options

A

ware; ware

ware; ware

B

wear; ware

wear; ware

C

ware; wear

ware; wear

D

wear; wear

wear; wear


Solution:

Correct Answer:

C

ware; wear

ware; wear


Question 196


Sentences of a paragraph are given below in jumbled order. Arrange the sentences in the correct order to form a meaningful and coherent paragraph.

A.The slogan ‘Jai Jawan’ and ‘Jai Kisan’ was first coined by Lal Bahadur Shastri immediately after the 1965 war.
B.This was coined to honour the two biggest pillars of Indian democracy: the soldiers and the farmers.
C.It seems once again, it is time for India to relook into its policies pertaining to soldiers and farmers
D.He strongly believed that soldiers and farmers were the backbone of Indian society and deserved to be honoured.

Sentences of a paragraph are given below in jumbled order. Arrange the sentences in the correct order to form a meaningful and coherent paragraph.

A.The slogan ‘Jai Jawan’ and ‘Jai Kisan’ was first coined by Lal Bahadur Shastri immediately after the 1965 war.
B.This was coined to honour the two biggest pillars of Indian democracy: the soldiers and the farmers.
C.It seems once again, it is time for India to relook into its policies pertaining to soldiers and farmers
D.He strongly believed that soldiers and farmers were the backbone of Indian society and deserved to be honoured.

Options

A

ABDC

ABDC

B

ADCB

ADCB

C

ABCD

ABCD

D

ACBD

ACBD


Solution:

Correct Answer:

A

ABDC

ABDC


Question 197


Select the most appropriate option to fill in the blank.

Let us meet at the coffeehouse. We can sit and have a chat __________ a cup of coffee.

Select the most appropriate option to fill in the blank.

Let us meet at the coffeehouse. We can sit and have a chat __________ a cup of coffee.

Options

A

on

on

B

above

above

C

over

over

D

at

at


Solution:

Correct Answer:

C

over

over


Question 198


Select the option that expresses the given sentence in indirect speech.

She said, “I sold my car myself.”

Select the option that expresses the given sentence in indirect speech.

She said, “I sold my car myself.”

Options

A

She said that I sold my car myself.

She said that I sold my car myself.

B

She said that she had sold her car herself.

She said that she had sold her car herself.

C

She said that I sold her car myself.

She said that I sold her car myself.

D

She said that she had sold my car herself.

She said that she had sold my car herself.


Solution:

Correct Answer:

B

She said that she had sold her car herself.

She said that she had sold her car herself.


Question 199


Select the most appropriate option to fill in the blank.

She suddenly remembered that she ________ her house before leaving for work.

Select the most appropriate option to fill in the blank.

She suddenly remembered that she ________ her house before leaving for work.

Options

A

does not lock

does not lock

B

has not locked

has not locked

C

had not locked

had not locked

D

was not locking

was not locking


Solution:

Correct Answer:

C

had not locked

had not locked


Question 200


Select the most appropriate option to fill in the blank.

Even now, many of us _________ the true meaning of freedom of speech.

Select the most appropriate option to fill in the blank.

Even now, many of us _________ the true meaning of freedom of speech.

Options

A

are not understanding

are not understanding

B

have not understood

have not understood

C

had not understood

had not understood

D

were not understanding

were not understanding


Solution:

Correct Answer:

B

have not understood

have not understood


Question 201


Select the most appropriate option that collocates with the word ‘tears’ to fill in the blank.

When she heard the news of her brother’s martyrdom, she _______ into tears.

Select the most appropriate option that collocates with the word ‘tears’ to fill in the blank.

When she heard the news of her brother’s martyrdom, she _______ into tears.

Options

A

spurt

spurt

B

rushed

rushed

C

gushed

gushed

D

burst

burst


Solution:

Correct Answer:

D

burst

burst


Question 202


Select the most appropriate option to fill in the blank.

A __________ of doves is nesting on my mango tree nowadays.

Select the most appropriate option to fill in the blank.

A __________ of doves is nesting on my mango tree nowadays.

Options

A

pare

pare

B

pair

pair

C

pear

pear

D

peer

peer


Solution:

Correct Answer:

B

pair

pair


Question 203


Select the most appropriate option that can substitute the underlined word in the given sentence. If there is no need to substitute it, select ‘No substitution required’.

We had given the patient the first support before the doctor arrived.

Select the most appropriate option that can substitute the underlined word in the given sentence. If there is no need to substitute it, select ‘No substitution required’.

We had given the patient the first support before the doctor arrived.

Options

A

No substitution required

No substitution required

B

benefit

benefit

C

aid

aid

D

help

help


Solution:

Correct Answer:

C

aid

aid


Question 204


Select the most appropriate ANTONYM of the given word.

Placate

Select the most appropriate ANTONYM of the given word.

Placate

Options

A

Annoy

Annoy

B

Humour

Humour

C

Comfort

Comfort

D

Indulge

Indulge


Solution:

Correct Answer:

A

Annoy

Annoy


Question 205


Select the correct punctuation mark to fill in the blank.

The meeting was chaired by Mr. Rao __________ the MD of our company

Select the correct punctuation mark to fill in the blank.

The meeting was chaired by Mr. Rao __________ the MD of our company

Options

A

colon (:)

colon (:)

B

dash (—)

dash (—)

C

comma (,)

comma (,)

D

semicolon (;)

semicolon (;)


Solution:

Correct Answer:

C

comma (,)

comma (,)


Question 206


Select the most appropriate synonym of the given word.

Dainty

Select the most appropriate synonym of the given word.

Dainty

Options

A

Tough

Tough

B

Nervous

Nervous

C

Annoyed

Annoyed

D

Fragile

Fragile


Solution:

Correct Answer:

D

Fragile

Fragile


Question 207


Select the most appropriate option to fill in the blank.

It’s so cold on the higher reaches of Ladakh that some soldiers get frost-bite in their __________.

Select the most appropriate option to fill in the blank.

It’s so cold on the higher reaches of Ladakh that some soldiers get frost-bite in their __________.

Options

A

toss

toss

B

twos

twos

C

toes

toes

D

tows

tows


Solution:

Correct Answer:

C

toes

toes


Question 208


Four sentences of a paragraph are given below in jumbled order. Arrange the sentences in the correct order to form a meaningful and coherent paragraph.

(A) In India, particularly in villages, even a few decades back, women without children were looked down upon.
(B) Such women were not invited for naming ceremonies.
(C) Nobody understood the hurt and trauma they underwent.
(D) And they were taunted as barren women.

Four sentences of a paragraph are given below in jumbled order. Arrange the sentences in the correct order to form a meaningful and coherent paragraph.

(A) In India, particularly in villages, even a few decades back, women without children were looked down upon.
(B) Such women were not invited for naming ceremonies.
(C) Nobody understood the hurt and trauma they underwent.
(D) And they were taunted as barren women.

Options

A

ADCB

ADCB

B

ABCD

ABCD

C

ACBD

ACBD

D

ABDC

ABDC


Solution:

Correct Answer:

D

ABDC

ABDC


Question 209


Select the most appropriate option to fill in the blank.

The speaker was least perturbed by the disturbance in the front rows and __________ with his speech.

Select the most appropriate option to fill in the blank.

The speaker was least perturbed by the disturbance in the front rows and __________ with his speech.

Options

A

carried away

carried away

B

carried on

carried on

C

carried off

carried off

D

carried out

carried out


Solution:

Correct Answer:

B

carried on

carried on


Question 210


Select the most appropriate meaning of the given idiom.

Rose-coloured glasses

Select the most appropriate meaning of the given idiom.

Rose-coloured glasses

Options

A

An unduly positive outlook on life

An unduly positive outlook on life

B

An outdated thing

An outdated thing

C

A dim view of things

A dim view of things

D

An event to remember

An event to remember


Solution:

Correct Answer:

A

An unduly positive outlook on life

An unduly positive outlook on life


Question 211


Select the most appropriate option to fill in the blank.

My mother enjoys __________ Turkish series on YouTube.

Select the most appropriate option to fill in the blank.

My mother enjoys __________ Turkish series on YouTube.

Options

A

watch

watch

B

watched

watched

C

watching

watching

D

to watch

to watch


Solution:

Correct Answer:

C

watching

watching


Question 212


Select the option that expresses the given sentence in active voice.

Captain Vikram Batra was awarded the Param Vir Chakra posthumously.

Select the option that expresses the given sentence in active voice.

Captain Vikram Batra was awarded the Param Vir Chakra posthumously.

Options

A

The Param Vir Chakra is being awarded to Captain Vikram Batra posthumously.

The Param Vir Chakra is being awarded to Captain Vikram Batra posthumously.

B

The Param Vir Chakra will be awarded to Captain Vikram Batra posthumously.

The Param Vir Chakra will be awarded to Captain Vikram Batra posthumously.

C

The Param Vir Chakra has been awarded to Captain Vikram Batra posthumously.

The Param Vir Chakra has been awarded to Captain Vikram Batra posthumously.

D

The Param Vir Chakra was awarded to Captain Vikram Batra posthumously.

The Param Vir Chakra was awarded to Captain Vikram Batra posthumously.


Solution:

Correct Answer:

D

The Param Vir Chakra was awarded to Captain Vikram Batra posthumously.

The Param Vir Chakra was awarded to Captain Vikram Batra posthumously.


Question 213


Select the most appropriate option to fill in the blank.

The __________ indication of his high rank were five stars in each of his shoulder straps.

Select the most appropriate option to fill in the blank.

The __________ indication of his high rank were five stars in each of his shoulder straps.

Options

A

only

only

B

just

just

C

little

little

D

much

much


Solution:

Correct Answer:

A

only

only


Question 214


Select the most appropriate option to fill in the blank.

Mother made Avika __________ her essay before submitting it.

Select the most appropriate option to fill in the blank.

Mother made Avika __________ her essay before submitting it.

Options

A

editing

editing

B

edited

edited

C

edit

edit

D

to edit

to edit


Solution:

Correct Answer:

C

edit

edit


Question 215


Select the most appropriate option that collocates with the word ‘pain’ to fill in the blank.

He suffered with _______ pain in his head after his car accident.

Select the most appropriate option that collocates with the word ‘pain’ to fill in the blank.

He suffered with _______ pain in his head after his car accident.

Options

A

strong

strong

B

robust

robust

C

heavy

heavy

D

excruciating

excruciating


Solution:

Correct Answer:

D

excruciating

excruciating


Question 216


Select the option that expresses the given sentence in direct speech.

The teacher told the students that most of them would be leaving school at the end of the session.

Select the option that expresses the given sentence in direct speech.

The teacher told the students that most of them would be leaving school at the end of the session.

Options

A

The teacher said to the students, “Most of them will leave school at the end of the session.”

The teacher said to the students, “Most of them will leave school at the end of the session.”

B

The teacher said to the students, “Most of you will be leaving school at the end of the session.”

The teacher said to the students, “Most of you will be leaving school at the end of the session.”

C

The teacher said to the students, “Most of you would have left school by the end of the session.”

The teacher said to the students, “Most of you would have left school by the end of the session.”

D

The teacher said to the students, “Most of them would be leaving school at the end of the session.”

The teacher said to the students, “Most of them would be leaving school at the end of the session.”


Solution:

Correct Answer:

B

The teacher said to the students, “Most of you will be leaving school at the end of the session.”

The teacher said to the students, “Most of you will be leaving school at the end of the session.”


Question 217


Select the correctly spelt word to fill in the blank.

As a Chartered Accountant, his ______ comprises big business people of the town.

Select the correctly spelt word to fill in the blank.

As a Chartered Accountant, his ______ comprises big business people of the town.

Options

A

clientele

clientele

B

cliantale

cliantale

C

clientale

clientale

D

cliantele

cliantele


Solution:

Correct Answer:

A

clientele

clientele


Question 218


Select the most appropriate option to fill in the blanks.

When I was young, I __________ to school on my bike on a dirt __________.

Select the most appropriate option to fill in the blanks.

When I was young, I __________ to school on my bike on a dirt __________.

Options

A

rode; road

rode; road

B

road; road

road; road

C

road; rode

road; rode

D

rode; rode

rode; rode


Solution:

Correct Answer:

A

rode; road

rode; road


Question 219


Select the sentence with the correct punctuation and capitalisation.

Select the sentence with the correct punctuation and capitalisation.

Options

A

“What’s the matter, Avika?” asked Rahul.

“What’s the matter, Avika?” asked Rahul.

B

“What’s the matter? Avika.” asked Rahul.

“What’s the matter? Avika.” asked Rahul.

C

“What’s the matter avika?” asked Rahul.

“What’s the matter avika?” asked Rahul.

D

“Whats the matter, Avika,” asked Rahul.

“Whats the matter, Avika,” asked Rahul.


Solution:

Correct Answer:

A

“What’s the matter, Avika?” asked Rahul.

“What’s the matter, Avika?” asked Rahul.


Question 220


Select the most appropriate option to fill in the blank.

They were armed __________ knives and swords.

Select the most appropriate option to fill in the blank.

They were armed __________ knives and swords.

Options

A

by

by

B

from

from

C

with

with

D

to

to


Solution:

Correct Answer:

C

with

with


Question 221


Comprehension:
In the following passage some words have been deleted. Read the passage carefully and select the most appropriate option to fill in each blank.

The James Webb Telescope has beamed back the (1)_________ pictures of Neptune, the last planet in our Solar System, in (2)__________ 30 years. The telescope has revealed the icy giant in a new light, (3)_________ 4.3 billion kilometres away from Earth, at the (4)__________ of the Solar System.
The telescope has picked up a crisp view of the rings of Neptune, (5)_________ had last been seen only in pictures taken by the Voyager 2 spacecraft when it conducted a flyby way back in 1989. Since then, the Voyager spacecraft is flying in interstellar space, millions of kilometres away from Neptune where the Sun’s impact ends.
SubQuestion No : 46
Select the most appropriate option to fill in the blank no. 1.

Comprehension:
In the following passage some words have been deleted. Read the passage carefully and select the most appropriate option to fill in each blank.

The James Webb Telescope has beamed back the (1)_________ pictures of Neptune, the last planet in our Solar System, in (2)__________ 30 years. The telescope has revealed the icy giant in a new light, (3)_________ 4.3 billion kilometres away from Earth, at the (4)__________ of the Solar System.
The telescope has picked up a crisp view of the rings of Neptune, (5)_________ had last been seen only in pictures taken by the Voyager 2 spacecraft when it conducted a flyby way back in 1989. Since then, the Voyager spacecraft is flying in interstellar space, millions of kilometres away from Neptune where the Sun’s impact ends.
SubQuestion No : 46
Select the most appropriate option to fill in the blank no. 1.

Options

A

clear

clear

B

more clear

more clear

C

clearer

clearer

D

clearest

clearest


Solution:

Correct Answer:

D

clearest

clearest


Question 222


Comprehension:
In the following passage some words have been deleted. Read the passage carefully and select the most appropriate option to fill in each blank.

The James Webb Telescope has beamed back the (1)_________ pictures of Neptune, the last planet in our Solar System, in (2)__________ 30 years. The telescope has revealed the icy giant in a new light, (3)_________ 4.3 billion kilometres away from Earth, at the (4)__________ of the Solar System.
The telescope has picked up a crisp view of the rings of Neptune, (5)_________ had last been seen only in pictures taken by the Voyager 2 spacecraft when it conducted a flyby way back in 1989. Since then, the Voyager spacecraft is flying in interstellar space, millions of kilometres away from Neptune where the Sun’s impact ends.
SubQuestion No : 47
Select the most appropriate option to fill in the blank no. 2.

Comprehension:
In the following passage some words have been deleted. Read the passage carefully and select the most appropriate option to fill in each blank.

The James Webb Telescope has beamed back the (1)_________ pictures of Neptune, the last planet in our Solar System, in (2)__________ 30 years. The telescope has revealed the icy giant in a new light, (3)_________ 4.3 billion kilometres away from Earth, at the (4)__________ of the Solar System.
The telescope has picked up a crisp view of the rings of Neptune, (5)_________ had last been seen only in pictures taken by the Voyager 2 spacecraft when it conducted a flyby way back in 1989. Since then, the Voyager spacecraft is flying in interstellar space, millions of kilometres away from Neptune where the Sun’s impact ends.
SubQuestion No : 47
Select the most appropriate option to fill in the blank no. 2.

Options

A

on

on

B

quite

quite

C

over

over

D

rather

rather


Solution:

Correct Answer:

C

over

over


Question 223


Comprehension:
In the following passage some words have been deleted. Read the passage carefully and select the most appropriate option to fill in each blank.

The James Webb Telescope has beamed back the (1)_________ pictures of Neptune, the last planet in our Solar System, in (2)__________ 30 years. The telescope has revealed the icy giant in a new light, (3)_________ 4.3 billion kilometres away from Earth, at the (4)__________ of the Solar System.
The telescope has picked up a crisp view of the rings of Neptune, (5)_________ had last been seen only in pictures taken by the Voyager 2 spacecraft when it conducted a flyby way back in 1989. Since then, the Voyager spacecraft is flying in interstellar space, millions of kilometres away from Neptune where the Sun’s impact ends.
SubQuestion No : 48
Select the most appropriate option to fill in the blank no. 3.

Comprehension:
In the following passage some words have been deleted. Read the passage carefully and select the most appropriate option to fill in each blank.

The James Webb Telescope has beamed back the (1)_________ pictures of Neptune, the last planet in our Solar System, in (2)__________ 30 years. The telescope has revealed the icy giant in a new light, (3)_________ 4.3 billion kilometres away from Earth, at the (4)__________ of the Solar System.
The telescope has picked up a crisp view of the rings of Neptune, (5)_________ had last been seen only in pictures taken by the Voyager 2 spacecraft when it conducted a flyby way back in 1989. Since then, the Voyager spacecraft is flying in interstellar space, millions of kilometres away from Neptune where the Sun’s impact ends.
SubQuestion No : 48
Select the most appropriate option to fill in the blank no. 3.

Options

A

hanged

hanged

B

hang

hang

C

to hang

to hang

D

hanging

hanging


Solution:

Correct Answer:

D

hanging

hanging


Question 224


Comprehension:
In the following passage some words have been deleted. Read the passage carefully and select the most appropriate option to fill in each blank.

The James Webb Telescope has beamed back the (1)_________ pictures of Neptune, the last planet in our Solar System, in (2)__________ 30 years. The telescope has revealed the icy giant in a new light, (3)_________ 4.3 billion kilometres away from Earth, at the (4)__________ of the Solar System.
The telescope has picked up a crisp view of the rings of Neptune, (5)_________ had last been seen only in pictures taken by the Voyager 2 spacecraft when it conducted a flyby way back in 1989. Since then, the Voyager spacecraft is flying in interstellar space, millions of kilometres away from Neptune where the Sun’s impact ends.
SubQuestion No : 49
Select the most appropriate option to fill in the blank no. 4.

Comprehension:
In the following passage some words have been deleted. Read the passage carefully and select the most appropriate option to fill in each blank.

The James Webb Telescope has beamed back the (1)_________ pictures of Neptune, the last planet in our Solar System, in (2)__________ 30 years. The telescope has revealed the icy giant in a new light, (3)_________ 4.3 billion kilometres away from Earth, at the (4)__________ of the Solar System.
The telescope has picked up a crisp view of the rings of Neptune, (5)_________ had last been seen only in pictures taken by the Voyager 2 spacecraft when it conducted a flyby way back in 1989. Since then, the Voyager spacecraft is flying in interstellar space, millions of kilometres away from Neptune where the Sun’s impact ends.
SubQuestion No : 49
Select the most appropriate option to fill in the blank no. 4.

Options

A

edge

edge

B

side

side

C

back

back

D

bank

bank


Solution:

Correct Answer:

A

edge

edge


Question 225


Comprehension:
In the following passage some words have been deleted. Read the passage carefully and select the most appropriate option to fill in each blank.

The James Webb Telescope has beamed back the (1)_________ pictures of Neptune, the last planet in our Solar System, in (2)__________ 30 years. The telescope has revealed the icy giant in a new light, (3)_________ 4.3 billion kilometres away from Earth, at the (4)__________ of the Solar System.
The telescope has picked up a crisp view of the rings of Neptune, (5)_________ had last been seen only in pictures taken by the Voyager 2 spacecraft when it conducted a flyby way back in 1989. Since then, the Voyager spacecraft is flying in interstellar space, millions of kilometres away from Neptune where the Sun’s impact ends.
SubQuestion No : 50
Select the most appropriate option to fill in the blank no. 5.

Comprehension:
In the following passage some words have been deleted. Read the passage carefully and select the most appropriate option to fill in each blank.

The James Webb Telescope has beamed back the (1)_________ pictures of Neptune, the last planet in our Solar System, in (2)__________ 30 years. The telescope has revealed the icy giant in a new light, (3)_________ 4.3 billion kilometres away from Earth, at the (4)__________ of the Solar System.
The telescope has picked up a crisp view of the rings of Neptune, (5)_________ had last been seen only in pictures taken by the Voyager 2 spacecraft when it conducted a flyby way back in 1989. Since then, the Voyager spacecraft is flying in interstellar space, millions of kilometres away from Neptune where the Sun’s impact ends.
SubQuestion No : 50
Select the most appropriate option to fill in the blank no. 5.

Options

A

who

who

B

what

what

C

whom

whom

D

which

which


Solution:

Correct Answer:

D

which

which


Question 226


Comprehension:
Read the following passage and answer the questions that follow.

The arrival of eight cheetahs in Gwalior, India on 17 September was a huge moment for a country where the animal was declared extinct back in 1952 due to loss of habitat and hunting. The cheetahs travelled from Namibia in a specially painted aircraft with a tiger’s face on it and the special Boeing 747 jet quickly became a sensation on social media. However, contrary to popular sentiment and some media reports, it was not painted for the occasion.
According to Newschecker, the plane was actually painted back in 2015 by Russian airline Transaero. The image on the plane was that of a Siberian Tiger and the purpose behind the painting was to increase awareness about the animals. The report further stated that the painting was a collaboration between the airlines and Amur Tiger Centre—an initiative signed by Russian President Vladimir Putin.
The first appearance of the flight with the tiger painting on its nose came in June 2015 at the Moscow Vnukovo airport. However, as Transaero went bankrupt, the plane was sent to storage in Spain before it was purchased by Sharjah-based Aquiline International in 2021 and leased it to Terra Avia of Moldova.
As a result, it was either pure coincidence or a planned strategy to use the specific flight for the transfer of cheetahs to India but the fact that it was specially painted for the occasion was not true.
The other possible explanation, according to Newschecker, was that the cheetahs needed to be transported in a jumbo-sized passenger plane and this was one of the few available for that purpose
The aircraft was arranged by Dubai-based Action Aviation for the project.
SubQuestion No : 51
What kind of a passage is it?

Comprehension:
Read the following passage and answer the questions that follow.

The arrival of eight cheetahs in Gwalior, India on 17 September was a huge moment for a country where the animal was declared extinct back in 1952 due to loss of habitat and hunting. The cheetahs travelled from Namibia in a specially painted aircraft with a tiger’s face on it and the special Boeing 747 jet quickly became a sensation on social media. However, contrary to popular sentiment and some media reports, it was not painted for the occasion.
According to Newschecker, the plane was actually painted back in 2015 by Russian airline Transaero. The image on the plane was that of a Siberian Tiger and the purpose behind the painting was to increase awareness about the animals. The report further stated that the painting was a collaboration between the airlines and Amur Tiger Centre—an initiative signed by Russian President Vladimir Putin.
The first appearance of the flight with the tiger painting on its nose came in June 2015 at the Moscow Vnukovo airport. However, as Transaero went bankrupt, the plane was sent to storage in Spain before it was purchased by Sharjah-based Aquiline International in 2021 and leased it to Terra Avia of Moldova.
As a result, it was either pure coincidence or a planned strategy to use the specific flight for the transfer of cheetahs to India but the fact that it was specially painted for the occasion was not true.
The other possible explanation, according to Newschecker, was that the cheetahs needed to be transported in a jumbo-sized passenger plane and this was one of the few available for that purpose
The aircraft was arranged by Dubai-based Action Aviation for the project.
SubQuestion No : 51
What kind of a passage is it?

Options

A

A government explanation

A government explanation

B

A review article

A review article

C

An investigative report

An investigative report

D

A news item

A news item


Solution:

Correct Answer:

D

A news item

A news item


Question 227


Comprehension:
Read the following passage and answer the questions that follow.

The arrival of eight cheetahs in Gwalior, India on 17 September was a huge moment for a country where the animal was declared extinct back in 1952 due to loss of habitat and hunting. The cheetahs travelled from Namibia in a specially painted aircraft with a tiger’s face on it and the special Boeing 747 jet quickly became a sensation on social media. However, contrary to popular sentiment and some media reports, it was not painted for the occasion.
According to Newschecker, the plane was actually painted back in 2015 by Russian airline Transaero. The image on the plane was that of a Siberian Tiger and the purpose behind the painting was to increase awareness about the animals. The report further stated that the painting was a collaboration between the airlines and Amur Tiger Centre—an initiative signed by Russian President Vladimir Putin.
The first appearance of the flight with the tiger painting on its nose came in June 2015 at the Moscow Vnukovo airport. However, as Transaero went bankrupt, the plane was sent to storage in Spain before it was purchased by Sharjah-based Aquiline International in 2021 and leased it to Terra Avia of Moldova.
As a result, it was either pure coincidence or a planned strategy to use the specific flight for the transfer of cheetahs to India but the fact that it was specially painted for the occasion was not true.
The other possible explanation, according to Newschecker, was that the cheetahs needed to be transported in a jumbo-sized passenger plane and this was one of the few available for that purpose
The aircraft was arranged by Dubai-based Action Aviation for the project.
SubQuestion No : 52
What is the main theme of the passage?

Comprehension:
Read the following passage and answer the questions that follow.

The arrival of eight cheetahs in Gwalior, India on 17 September was a huge moment for a country where the animal was declared extinct back in 1952 due to loss of habitat and hunting. The cheetahs travelled from Namibia in a specially painted aircraft with a tiger’s face on it and the special Boeing 747 jet quickly became a sensation on social media. However, contrary to popular sentiment and some media reports, it was not painted for the occasion.
According to Newschecker, the plane was actually painted back in 2015 by Russian airline Transaero. The image on the plane was that of a Siberian Tiger and the purpose behind the painting was to increase awareness about the animals. The report further stated that the painting was a collaboration between the airlines and Amur Tiger Centre—an initiative signed by Russian President Vladimir Putin.
The first appearance of the flight with the tiger painting on its nose came in June 2015 at the Moscow Vnukovo airport. However, as Transaero went bankrupt, the plane was sent to storage in Spain before it was purchased by Sharjah-based Aquiline International in 2021 and leased it to Terra Avia of Moldova.
As a result, it was either pure coincidence or a planned strategy to use the specific flight for the transfer of cheetahs to India but the fact that it was specially painted for the occasion was not true.
The other possible explanation, according to Newschecker, was that the cheetahs needed to be transported in a jumbo-sized passenger plane and this was one of the few available for that purpose
The aircraft was arranged by Dubai-based Action Aviation for the project.
SubQuestion No : 52
What is the main theme of the passage?

Options

A

The tiger-faced airplane

The tiger-faced airplane

B

The bankrupt airlines

The bankrupt airlines

C

The transportation of cheetahs

The transportation of cheetahs

D

The arrival of eight cheetahs

The arrival of eight cheetahs


Solution:

Correct Answer:

A

The tiger-faced airplane

The tiger-faced airplane


Question 228


Comprehension:
Read the following passage and answer the questions that follow.

The arrival of eight cheetahs in Gwalior, India on 17 September was a huge moment for a country where the animal was declared extinct back in 1952 due to loss of habitat and hunting. The cheetahs travelled from Namibia in a specially painted aircraft with a tiger’s face on it and the special Boeing 747 jet quickly became a sensation on social media. However, contrary to popular sentiment and some media reports, it was not painted for the occasion.
According to Newschecker, the plane was actually painted back in 2015 by Russian airline Transaero. The image on the plane was that of a Siberian Tiger and the purpose behind the painting was to increase awareness about the animals. The report further stated that the painting was a collaboration between the airlines and Amur Tiger Centre—an initiative signed by Russian President Vladimir Putin.
The first appearance of the flight with the tiger painting on its nose came in June 2015 at the Moscow Vnukovo airport. However, as Transaero went bankrupt, the plane was sent to storage in Spain before it was purchased by Sharjah-based Aquiline International in 2021 and leased it to Terra Avia of Moldova.
As a result, it was either pure coincidence or a planned strategy to use the specific flight for the transfer of cheetahs to India but the fact that it was specially painted for the occasion was not true.
The other possible explanation, according to Newschecker, was that the cheetahs needed to be transported in a jumbo-sized passenger plane and this was one of the few available for that purpose
The aircraft was arranged by Dubai-based Action Aviation for the project.
SubQuestion No : 53
What was the purpose of painting a tiger’s face on a plane?

Comprehension:
Read the following passage and answer the questions that follow.

The arrival of eight cheetahs in Gwalior, India on 17 September was a huge moment for a country where the animal was declared extinct back in 1952 due to loss of habitat and hunting. The cheetahs travelled from Namibia in a specially painted aircraft with a tiger’s face on it and the special Boeing 747 jet quickly became a sensation on social media. However, contrary to popular sentiment and some media reports, it was not painted for the occasion.
According to Newschecker, the plane was actually painted back in 2015 by Russian airline Transaero. The image on the plane was that of a Siberian Tiger and the purpose behind the painting was to increase awareness about the animals. The report further stated that the painting was a collaboration between the airlines and Amur Tiger Centre—an initiative signed by Russian President Vladimir Putin.
The first appearance of the flight with the tiger painting on its nose came in June 2015 at the Moscow Vnukovo airport. However, as Transaero went bankrupt, the plane was sent to storage in Spain before it was purchased by Sharjah-based Aquiline International in 2021 and leased it to Terra Avia of Moldova.
As a result, it was either pure coincidence or a planned strategy to use the specific flight for the transfer of cheetahs to India but the fact that it was specially painted for the occasion was not true.
The other possible explanation, according to Newschecker, was that the cheetahs needed to be transported in a jumbo-sized passenger plane and this was one of the few available for that purpose
The aircraft was arranged by Dubai-based Action Aviation for the project.
SubQuestion No : 53
What was the purpose of painting a tiger’s face on a plane?

Options

A

It was specially painted as the plane was going to transport cheetahs.

It was specially painted as the plane was going to transport cheetahs.

B

It was painted on the orders from the Russian President.

It was painted on the orders from the Russian President.

C

It was painted to increase awareness about animals.

It was painted to increase awareness about animals.

D

It was painted to indicate that the plane was an animal carrier.

It was painted to indicate that the plane was an animal carrier.


Solution:

Correct Answer:

C

It was painted to increase awareness about animals.

It was painted to increase awareness about animals.


Question 229


Comprehension:
Read the following passage and answer the questions that follow.

The arrival of eight cheetahs in Gwalior, India on 17 September was a huge moment for a country where the animal was declared extinct back in 1952 due to loss of habitat and hunting. The cheetahs travelled from Namibia in a specially painted aircraft with a tiger’s face on it and the special Boeing 747 jet quickly became a sensation on social media. However, contrary to popular sentiment and some media reports, it was not painted for the occasion.
According to Newschecker, the plane was actually painted back in 2015 by Russian airline Transaero. The image on the plane was that of a Siberian Tiger and the purpose behind the painting was to increase awareness about the animals. The report further stated that the painting was a collaboration between the airlines and Amur Tiger Centre—an initiative signed by Russian President Vladimir Putin.
The first appearance of the flight with the tiger painting on its nose came in June 2015 at the Moscow Vnukovo airport. However, as Transaero went bankrupt, the plane was sent to storage in Spain before it was purchased by Sharjah-based Aquiline International in 2021 and leased it to Terra Avia of Moldova.
As a result, it was either pure coincidence or a planned strategy to use the specific flight for the transfer of cheetahs to India but the fact that it was specially painted for the occasion was not true.
The other possible explanation, according to Newschecker, was that the cheetahs needed to be transported in a jumbo-sized passenger plane and this was one of the few available for that purpose
The aircraft was arranged by Dubai-based Action Aviation for the project.
SubQuestion No : 54
What did the people and media in India think about the tiger-faced plane that transported the cheetahs from Namibia to India?

Comprehension:
Read the following passage and answer the questions that follow.

The arrival of eight cheetahs in Gwalior, India on 17 September was a huge moment for a country where the animal was declared extinct back in 1952 due to loss of habitat and hunting. The cheetahs travelled from Namibia in a specially painted aircraft with a tiger’s face on it and the special Boeing 747 jet quickly became a sensation on social media. However, contrary to popular sentiment and some media reports, it was not painted for the occasion.
According to Newschecker, the plane was actually painted back in 2015 by Russian airline Transaero. The image on the plane was that of a Siberian Tiger and the purpose behind the painting was to increase awareness about the animals. The report further stated that the painting was a collaboration between the airlines and Amur Tiger Centre—an initiative signed by Russian President Vladimir Putin.
The first appearance of the flight with the tiger painting on its nose came in June 2015 at the Moscow Vnukovo airport. However, as Transaero went bankrupt, the plane was sent to storage in Spain before it was purchased by Sharjah-based Aquiline International in 2021 and leased it to Terra Avia of Moldova.
As a result, it was either pure coincidence or a planned strategy to use the specific flight for the transfer of cheetahs to India but the fact that it was specially painted for the occasion was not true.
The other possible explanation, according to Newschecker, was that the cheetahs needed to be transported in a jumbo-sized passenger plane and this was one of the few available for that purpose
The aircraft was arranged by Dubai-based Action Aviation for the project.
SubQuestion No : 54
What did the people and media in India think about the tiger-faced plane that transported the cheetahs from Namibia to India?

Options

A

It was the only suitable plane available for the purpose.

It was the only suitable plane available for the purpose.

B

It was a planned strategy of the government to acquire that special plane for the purpose.

It was a planned strategy of the government to acquire that special plane for the purpose.

C

It was a coincidence that the government got that plane for the purpose.

It was a coincidence that the government got that plane for the purpose.

D

The government has purposely got the plane painted for the occasion.

The government has purposely got the plane painted for the occasion.


Solution:

Correct Answer:

D

The government has purposely got the plane painted for the occasion.

The government has purposely got the plane painted for the occasion.


Question 230


Comprehension:
Read the following passage and answer the questions that follow.

The arrival of eight cheetahs in Gwalior, India on 17 September was a huge moment for a country where the animal was declared extinct back in 1952 due to loss of habitat and hunting. The cheetahs travelled from Namibia in a specially painted aircraft with a tiger’s face on it and the special Boeing 747 jet quickly became a sensation on social media. However, contrary to popular sentiment and some media reports, it was not painted for the occasion.
According to Newschecker, the plane was actually painted back in 2015 by Russian airline Transaero. The image on the plane was that of a Siberian Tiger and the purpose behind the painting was to increase awareness about the animals. The report further stated that the painting was a collaboration between the airlines and Amur Tiger Centre—an initiative signed by Russian President Vladimir Putin.
The first appearance of the flight with the tiger painting on its nose came in June 2015 at the Moscow Vnukovo airport. However, as Transaero went bankrupt, the plane was sent to storage in Spain before it was purchased by Sharjah-based Aquiline International in 2021 and leased it to Terra Avia of Moldova.
As a result, it was either pure coincidence or a planned strategy to use the specific flight for the transfer of cheetahs to India but the fact that it was specially painted for the occasion was not true.
The other possible explanation, according to Newschecker, was that the cheetahs needed to be transported in a jumbo-sized passenger plane and this was one of the few available for that purpose
The aircraft was arranged by Dubai-based Action Aviation for the project.
SubQuestion No : 55
Which statement is NOT true about the tiger-faced plane?

Comprehension:
Read the following passage and answer the questions that follow.

The arrival of eight cheetahs in Gwalior, India on 17 September was a huge moment for a country where the animal was declared extinct back in 1952 due to loss of habitat and hunting. The cheetahs travelled from Namibia in a specially painted aircraft with a tiger’s face on it and the special Boeing 747 jet quickly became a sensation on social media. However, contrary to popular sentiment and some media reports, it was not painted for the occasion.
According to Newschecker, the plane was actually painted back in 2015 by Russian airline Transaero. The image on the plane was that of a Siberian Tiger and the purpose behind the painting was to increase awareness about the animals. The report further stated that the painting was a collaboration between the airlines and Amur Tiger Centre—an initiative signed by Russian President Vladimir Putin.
The first appearance of the flight with the tiger painting on its nose came in June 2015 at the Moscow Vnukovo airport. However, as Transaero went bankrupt, the plane was sent to storage in Spain before it was purchased by Sharjah-based Aquiline International in 2021 and leased it to Terra Avia of Moldova.
As a result, it was either pure coincidence or a planned strategy to use the specific flight for the transfer of cheetahs to India but the fact that it was specially painted for the occasion was not true.
The other possible explanation, according to Newschecker, was that the cheetahs needed to be transported in a jumbo-sized passenger plane and this was one of the few available for that purpose
The aircraft was arranged by Dubai-based Action Aviation for the project.
SubQuestion No : 55
Which statement is NOT true about the tiger-faced plane?

Options

A

The plane was actually painted back in 2015 by Russian airline Transaero.

The plane was actually painted back in 2015 by Russian airline Transaero.

B

The plane was sent to the Indian government by the Transaero airlines.

The plane was sent to the Indian government by the Transaero airlines.

C

The painting was a collaboration between the airlines and Amur Tiger Centre.

The painting was a collaboration between the airlines and Amur Tiger Centre.

D

The image on the plane was that of a Siberian Tiger and not of Indian tiger.

The image on the plane was that of a Siberian Tiger and not of Indian tiger.


Solution:

Correct Answer:

B

The plane was sent to the Indian government by the Transaero airlines.

The plane was sent to the Indian government by the Transaero airlines.


Types of Web Hosting

21-Apr-2023 06:03:43 | BLOG


types of web hosting


Read More

How to choose best web hosting

27-Sep-2022 09:46:18 | BLOG


best hosting


Read More

A2 Hosting Review

27-Sep-2022 09:45:14 | BLOG


a2 hosting


Read More

HostPapa Review

27-Sep-2022 09:44:24 | BLOG


hostpapa


Read More

Dreamhost Review

27-Sep-2022 09:43:44 | BLOG


dreamhost


Read More

Hostgator Review

27-Sep-2022 09:43:02 | BLOG


hostgator


Read More

Hostinger Review

27-Sep-2022 09:42:05 | BLOG


Hostinger


Read More

inMotion Hosting Review

27-Sep-2022 09:41:15 | BLOG


inmotion


Read More